Download as pdf or txt
Download as pdf or txt
You are on page 1of 89

SURGERY

BY HAMOUD ALMUBKI
An approach to high-yield SMLE
surgery topics and questions

Telegram channel: https://t.me/SurgeryByHamoud

References: Schwartz, Washington Manual


of Surgery, UTD, Amboss, Aldilaijain course,
Thawaba course
Side note: before we begin I would like to explain the different
kind of questions used in the SMLE.

1) Initial - Next step - Most appropriate next step = what is


your action now in the full approach from A to Z starting with a
simple physical exam for instance and working your way up to an
invasive investigation. However, it has to be clinically relevant.

2) Most appropriate management - Most appropriate investigation


= the best choice for this specific case (you can skip a step) but it
doesn’t have to be the most diagnostic. This kind of question
requires nuance. It can be the next step, or it can be the gold
standard if the initial choice is not that useful, not necessarily the
very next step. Depends, so use your clinical judgment.

3) Most diagnostic - Most accurate - Gold standard - Definitive =


the very best choice, even if it’s not needed now. Straightforward.

A lot of questions in the Qbank are bad recalls, so keep this in


mind as you study.
Thyroid
First, we have to do a physical exam for assessment, followed by TSH and FT4,
then it depends on the TSH value.

Low TSH
Do a thyroid scan to check the uptake in a hyperthyroidism case
No uptake -> Subacute thyroiditis (has neck pain)
Diffuse -> Graves (also if positive antibody)
Nodular if single, multinodular goiter if multiple nodules.

Always initiate treatment for hot nodules with an antithyroid drug first to reach
euthyroid status (prevent thyroid storm), followed by RAI or surgery. Go with
near total thyroidectomy if compressive symptoms exist (such as dysphagia), or
if the patient failed medical treatment, or if they have exopthalmos. Medically, if
you had to choose between beta blockers and an antithyroid drug, go with beta
blockers first for symptomatic treatment. Pregnant in the first trimester?
Choose propylthiouracil.

Only choose FNA for cold solid nodules.

Normal or high TSH


Ultrasound to assess the nodule/s, then FNA nodules if the size is 1*1cm or
bigger. If both nodules are smaller than that, follow up with ultrasound.
Afterwards, it depends on the Bethesda classification for cold thyroid nodules:

1 - repeat FNA
2 - follow up with US
3 - repeat FNA
4 - hemithyroidectomy
5 - near total is preferred over hemithyroidectomy
6 - near total thyroidectomy

If a diagnosis of hypothyroidism is made, start levothyroxine and reassess after


6 weeks. Titrate the dose depending on the follow up visit.
If the TSH isn’t mentioned in the question, pretend that it’s normal. Always
follow the approach step by step. If the TSH is normal and there’s clinical
suspicion, repeat TSH unless there’s a history of radiofrequency ablation.
Thyroid cancer follow up:
1) Papillary and Follicular are followed up by Thyroglobulin
2) Medullary is followed up by Calcitonin

Most common thyroid cancer associated with autoimmune


thyroiditis is Lymphoma (not Lymphoblastic) followed by
Papillary.

Neck nodule
Thyroglossal cyst (central) moves with tongue protrusion.
Treated surgically.
Cystic hygroma (lateral) has clear lymphatic fluid. The initial
treatment is sclerotherapy, best is surgical.
1- 35 years old male patient underwent US to neck for another reason and
accidentally discover nodule in the right lobe, low TSH high T4, what is
the proper investigation?

A. Radioactive iodine scan.


B. Hemithyroidectomy.
C. FNA.
D. Follow up.

Low TSH case. In other words, it’s hyperthyroidism thus the next step is
a thyroid scan.

2- unilateral neck swelling in the RT side by investigations : hot


thyroid nodule remaining of the gland cold TSH is low, T3, T4 high
No LN enlargement (dx hyperthyroidism toxic nodule) What is the initial
Treatment?

A- antithyroid drug
B- RT thyroidectomy
C- Hemithyroidectomy
D- radioactive iodine

Achieve euthyroid status first with an antithyroid medication, then go for radioactive iodine,
or near total thyroidectomy if there is presence of eye symptoms or compressive symptoms.

3- A patient presented with history of hyperthyroidism and exophthalmos. TSH is low with
High T4 and T3. On thyroid scan there is increase uptake and suggestive of thyroid nodular
goiter
(right sides 1/2cm), which of the following is the best management?

A- Right hemithyroidectomy.
B- Subtotal thyroidectomy.
C- Near-total thyroidectomy. Near total thyroidectomy due to the presence of eye
D- Radioactive ablation. symptoms. Can't find near-total in the options? Go
with total.

4- A thyroid mass, TFT is not mentioned, US: large cystic mass with small solid
mass, what is the best next step in management?
A. FNA from cystic.
B. FNA from solid. As explained earlier, we only take samples from solid nodules. In
C. US. another recall, let’s say you had to choose between FNA cystic and
D. Thyroid scan. biopsy solid, go for biopsy solid.

5- 48 yo lady with diffuse goiter, high T4 low TSH, US show bilateral thyroid nodules , right
3x4 in size , left is 1x2 size what to do?
A. FNA both
B. FNA the larger one
C. Total thyroidectomy

The sizes are in CM, and they’re both bigger than 1*1cm, so we need to FNA both
for the risk of malignancy. If thyroid scan was in the choices, go for it since TSH
is low.
6- Let’s assume we had the same case with an US showing bilateral thyroid
nodules. The right nodule being 3x4mm in size, and the left is 1x2mm. Next?
A. FNA both
B. FNA the larger one
C. total thyroidectomy
D. Follow up with US

Because both are smaller than 1*1cm.

7- 3x4mm in size, and the other nodule is 1x2cm. Next?


A. FNA both
B. FNA the larger one
C. total thyroidectomy
D. US follow up

8- 25 years old female with thyroid nodule, TSH and T4 normal. FNA cytology done
classified as Bethesda 3. What is the most appropriate management?
A. Repeat FNA.
B. Lobectomy.
C. Levothyroxine. Side note: lobectomy = hemithyroidectomy.
D. Total thyroidectomy.

9- 40 years old male with left neck mass, thyroid ultrasound done and showed
1.5*2 cm cold mass, FNA cytology revealed suspicious follicular neoplasm,
which of the following the best initial management?

A. Left hemithyroidectomy.
B. Repeat FNA.
C. Prescribe thyroxine.
D. Radioactive ablation.

Follicular neoplasm = Bethesda 4, so go for hemithyroidectomy.

10- 45 years old female known case of hypothyroidism on levothyroxine with high TSH. The
patient is scheduled for elective surgery. What is the most appropriate management?
A. Increase levothyroxine dose.
B. Stop levothyroxine before the surgery.
C. Postpone the surgery until euthyroid is restored.
D. Proceed with the surgery and prescribe thyroxine after the surgery.

As explained earlier, never proceed with surgery until you achieve euthyroid status.

11- 55 years old patient was diagnosed with autoimmune thyroiditis, with a history of
progressive enlargement of the right lobe of the thyroid. FNA report: malignant cells. Which
of the following is most likely the type of thyroid malignancy?
A- Papillary.
B- Medullary.
C- Anaplastic.
D- Lymphoma.

Lymphoma. If not present, go with papillary. If you see lymphoblastic in the


options, then it’s wrong.
12- 32 years old male with neck mass and dysphagia, US done and showed mass 2*3 cm, FNA
cytology reveled hypercellular of large poorly cohesive spindle-like shape cells, labs showed high
calcitonin, Which of the following is the most likely diagnosis?
A. Papillary.
B. Medullary.
C. Anaplastic.
D. Follicular.

Calcitonin? Medullary thyroid cancer.

13- Middle age Pt with papillary thyroid cancer ,


planned for total thyroidectomy how to follow up ?
A- serial post op US
B- calcitonin
C- TSH , T3 , T4
D- Thyroglobulin

Papillary and follicular are followed up by thyroglobulin. But if it was medullary cancer?
Go with calcitonin.

14- 36 year-old female presented with left neck mass 2x2cm in posterior angle of
mandible. US: Normal thyroid, left large LN with cystic component. FNA: All smear
shows follicular thyroid What is the most likely diagnosis?

A. Metastatic thyroid cancer


B. Apparent thyroid
C. Ectopic thyroid
D. Thryoglossal cyst

This is a case of papillary thyroid cancer metastasizing to the lymph nodes.

15- 35 years old female underwent thyroid lobectomy for hot thyroid nodule 3x3,
8mm papillary will defined focus was found distant to the lesion with no lymph or
vascular invasion, what is the appropriate management?
A- Completion thyroidectomy.
B- Follow up 3 months.
C- RAI.
D- Lobectomy.

Since the size is only 8mm, simply follow up. We go with total thyroidectomy if it’s 1cm or
bigger.

16- A patient underwent thyroidectomy, immediately developed shortness of breath


and respiratory distress after extubation. Upon examination both vocal cord are in
semi-closed position. What is the best next step to secure the airway?

A- Cricothyrotomy
B- Re-intubate.
C- Nasal canula.
D- Bedside tracheostomy.

A case of bilateral vocal cord paralysis post-thyroidectomy. Requires


immediate reintubation.
17- A female patient underwent left thyroid lobectomy, post surgery she complaining of
sever shortness of breath and pointing to her neck, what is the next step?
A- Take her back to OR.
B- Bedside tracheostomy.
C- Nasal cannula.
D- Bedside wound exploration.

This question has a few different recalls when it comes to the wording in the answers. The
correct answer is opening the wound immediately due to the hematoma.

18- A patient post thyroidectomy was unable to maintain a high-pitched voice. Which of the
following is the injured nerve?
A. Superior laryngeal
B. Inferior laryngeal
C. Recurrent laryngeal
D. External laryngeal.

High pitched voice = superior laryngeal nerve.

19- A patient post thyroidectomy then developed hoarseness of voice and


aspiration. Which of the following is the injured nerve?
A. Superior laryngeal
B. Inferior laryngeal
C. Recurrent laryngeal
D. External laryngeal.

Hoarseness = recurrent laryngeal nerve.

20- 4 years old boy, developed large neck swelling on the left side. On exam it is large
10cm in size, US FNA showed clear lymphatic fluid
What is the appropriate management?
A- Surgery
B- Observation
C- Chemotherapy
D- Radiotherapy

This is a case of cystic hygroma, a lateral neck mass with clear lymphatic fluid.
Initial: sclerotherapy. Best: surgery

21- A child with painless neck nodule that moves up and down, moves up with
tongue protrusion. What is the most likely diagnosis?

A. Dermoid cyst.
B. Thyroid nodule.
C. Thyroglossal cyst.
D. Parathyroid nodule.

Midline mass and moves with tongue protrusion. Another recall asking
about management? Go with surgery.
22- Pregnant lady, not sure about the gestational age. Presenting with signs
and symptoms of hyperthyroidism. Which of the following is most contraindicated?
A- thyroidectomy.
B- radioactive iodine.
C- propranolol.
D- methimazole.

23- Patient admitted to the ICU due to pneumonia then developed thyroid
symptoms (hypo or hyperthyroid symptoms I don’t remember, but no symptoms
suggesting graves or subacute thyroiditis)
Labs:
TSH low
T3 low
T4 low
What’s the diagnosis?
A-Graves disease
B-Sick euthyroid sickness
C-Hashimoto thyroiditis Because of the lab values.
D-Subacute thyroiditis

24- Woman has controlled hypothyroidism on 175 mcg levothyroxine. In the last
3 months,
the
doctor raised the dose to 200 mcg. She is otherwise normal (see labs).
Labs:
T4 (normal)
T3 (normal)
TSH= 17 (high).
What is the best explanation for the lab findings?
A. Small dose. Normal T4 + High TSH = Poor compliance. T4 improves
B. Poor compliance. quickly by taking levothyroxine, but the TSH requires
C. Ectopic thyroid. daily compliance long term or it will continue to stay
D. Secondary hypothyroidism high. If the patient had actual good compliance and the
TSH is still high? Then we need to increase the dose 6
weeks after the last appointment.

25- case of hyperthyroidism and palpation. What will you do for her as initial management?
A. PTU
B. Methimazole
C. Propranolol
In real life, you would start both methimazole and propanolol at the same time. In the
SMLE though go for propanolol first for symptomatic relief.

26- Long case of a medullary thyroid cancer (diagnosis given) what is the appropriate
management?
A- Sub total thyroidectomy
B- Total thyroidectomy
C- Hemithyroidectomy

Follow the Bethesda classification. Case of thyroid cancer? Go for near


total thyroidectomy. If that’s not available in the options, go for total
thyroidectomy.
27- Case of an asthmatic patient complaining of shortness of breath in supine with
dysphagia, CT scan done and showed midline mass, what is the most likely diagnosis?
A. Lymphoma.
B. Thymoma.
C. Goiter.
D. Lung nodule.

A thyroid goiter that led to compressive symptoms (dysphagia). The


patient needs surgery.

28- A patient post thyroidectomy had arm spasm during blood pressure
measurement, what is your next step?

A- Give analgesia.
B- Take blood pressure again.
C- Check calcium level.
D- Administer Calcium Gluconate.

Hypocalcemia as a complication of thyroidectomy.

29- 19 year old pt found accidentally lump in her throat she did tsh was normal and did FNA
was inadequate;
a-Repeat FNA
b-Scan
c-Remove thyroid
d-Follow up

Inadequate FNA means the Bethesda score is 1, so you have to


repeat FNA now.
Breast
Triple assessment approach
The first step in evaluation is physical examination followed by imaging,
then biopsy. Go with ultrasound for patients under 40 years old, and
mammography for older patients. If the patient has a positive family
history of a first degree relative 10 years older, choose mammography
even if the patient is in their 30s.
Afterwards, go with core needle biopsy to confirm the diagnosis. In a
breast case, only choose FNA for cystic lesions.

BI-RAD
0 - additional imaging required
1 and 2 - annual screening
3 - follow up imaging in 6 months
4 and 5 - core biopsy
6 - surgery

Mammography screening is done annually in the range of 40 to 50 years


old. Afterwards, every two years according to the USPSTF.

A core needle biopsy showing intraductal hyperplasia? Go


with WLE.

Fibroadenoma -> oval shape, mobile mass, related to the menstrual


cycle. Treated with WLE.

Phyllode -> rapidly growing, not related to the menstrual cycle. Treated
with WLE if benign phyllode, or simple mastectomy if malignant, or
8*8cm+
Choose CT with contrast first in a malignant case for staging.

Intraductal papilloma -> most common cause of non-lactating nipple


discharge. Bloody.

Duct ectasia -> inverted nipple, green discharge. Both ectasia and
papilloma are treated with intraductal excision.

Fibrocystic changes -> multiple bilateral small masses, milky


discharge, painful.

Fat cyst/necrosis -> skin retraction overlying the breast in addition to


ecchymosis.

Montgomery follicles -> non-tender lumps confined to the areola.


Paget disease -> rare cancer. Unilateral involvement: 1)
Erythematous 2) Pruritis 3) Nipple destruction. Treated by
mastectomy with SLNB.

Ductal Carcinoma In Situ (DCIS) -> mastectomy with SLNB, or


WLE with radiation.

Lobular Carcinoma In Situ (LCIS) -> interval screening/close


surveillance.

Family history
BRCA is an autosomal dominant gene mutation associated
with an increased risk for early onset breast and ovarian
cancer. If suspected in first degree relatives, first screen the
affected patients with cancer to confirm, then screen the entire
family.

BRCA positive screening


1) Annual MRI at 25 years old.
2) Annual mammography at 30 years old.
3) Annual ovarian screening at 30 years old which
includes a pelvic US, and CA 125.

Skin tethering is a keyword for a patient with breast


cancer.
Atypia is the most significant risk factor.

Inflammation
An erythematous firm swelling is mastitis. It’s treated with
antibiotics.
However, if the patient has a fluctuant mass, of if there’s any
skin changes besides redness, then it’s an abscess.

Abscess
If one of the following is present then choose I+D Multiple,
5cm or more, thinned, ischemic or necrotic skin. Otherwise,
aspiration is enough.
30- 32 y/o women presented to clinic complaining of left breast pain and nipple bloody
discharge No family history of breast cancer Normal breast and lymph node examination
Which of the following is most appropriate test?
A- CT
B- MRI
C- US
D- Mammogram

Since the patient is younger than 40 years old, we go for an US.

31- 37 years old female her father has colon cancer when he was 55 and her mom had
breast cancer when she
was 43, asking about screening?
A.She should do mammogram annually
B.Start mammogram at 40
C.Start mammogram at 40 and colonoscopy at 55
D.Colonoscopy at 50

Because of family history, start mammography 10 years prior to


her mother’s onset of breast cancer. No family history? Wait until she is 40 years old.

32- 40 year old female presented with a left breast mass.


US and Mammogram showed a complex mass. What is the
next step in management?
A. Aspirate
B. Core needle biopsy
C. Excision
D. Follow up 12 months from now

Triple assessment approach. Imaging was done, so the next step is biopsy.

33- 55 years old female came with bilateral breast pain + bilateral green discharge from
multiple ducts. Imaging: Multiple dilated ducts, not suspicious. What to do?
A. us guided needle biopsy
B. Interval follow up imaging
C. Galactogram
D. MRI breast
In a typical breast case, after going for imaging the next step is biopsy.
However, this patient has physiologic bilateral nipple discharge which
could have a variety of different colors, but the most important keyword is
multiple ducts and bilateral. If it was unilateral + green = a clear case of
duct ectasia so only then we would choose biopsy.

34- A 35 year old lady presents with a left nipple bloody discharge, by imaging it was
suggestive of Intraductal papilloma. What to do next?
A. Left Central Duct excision
B. Observation
C. Interval follow up imaging
D. image guided biopsy

Bloody nipple discharge indicates a case of intraductal papilloma.


35- 35yo Female with RT bloody/or green nipple discharge mamo negative, us shows bilateral
duct dilation and something on Rt breast ddx(IDP,duct ectasia etc..) next?
A-duct excision
B-core needle biopsy
C-galactography
D-mastectomy

Bad recall, but it’s certainly a case of intraductal papilloma (bloody) rather than duct
ectasia (green). Although both are treated similarly, if we had a case of bilateral green
discharge then that’s physiological. In the case of papilloma? It can be bilateral or
unilateral.

36- 60 years old female came with bloody nipple discharge


Most appropriate steps to her management:
A- mammogram annually
B- start ultrasound
C- MRI

Due to her age it should be mammography now, not annually. If it comes like
this go with US. We only go with an MRI if both mammography and US are
negative.

37- 36 years old female came for routine check.


The report was as follows:US: can't remember the details)
Comment: BI-RADS III / propably benign
CT: multiple fibrous tissues with no calcifications
How will you nanage this patien?
A-Follow up after 6 months
B-Core biopsy
C-MRI breast

38- A woman with a left breast mass for 9months. Mammogram: speculated mass with
suspicious microcalcification and axillary lymph node involvement, BI-RAD V (probable
malignancy), next step?
A. Excisional biopsy
B. core Biopsy
C. Modified radical mastectomy

Core needle biopsy, also referred to as true-cut biopsy in some recalls. Be sure to
review the BI-RAD scores.

39- Patient presented with a mass in the breast, which was growing according to her in the
past several years, on Examiantion there was a 15x15 mass, upon doing fna It was a
“cystosarcoma phyllodes”
What is the appropriate management ?
A- chemo
B- radio
C- mastectomy
D- MRM

As explained at the start of this section, we go for a simple mastectomy due to the size.
40- women with 11*12 breast mass, examination showed no palpable LN. Core biopsy was
taken
and showed malignant phyllode tumor, what’s the next appropriate step?
a. WLE
b. PET scan
c. Chest CT without contrast
d. Simple mastectomy

As this is a case of malignant phyllodes, we need to go for chest CT with contrast


first to stage the cancer, but since the option here is without contrast go for simple
mastectomy.
41- 4.5cm malignant phyllodes management ?
A- WLE
B-mastectomy
In a malignant case, we either go for WLE with radiation or mastectomy. It
would be preferred to go for WLE with radiation due to the small size so
we can conserve the cosmetic appearance of the breast, but it’s missing
here.

42- 53y did mammogram now and was normal, when she should do again?
A-6m
B-1y
C-2y
D-3y
Mammography screening is done annually in the range of 40 to 50 years old.
Afterwards, every two years according to the USPSTF.

43- Female in her 47 , single, positive family history of breast cancer. Underwent routine
mammography which showed bilateral increased density and glandular pattern. Core needle
biopsy showed atypical ductal hyperplasia. What’s the appropriate management?
A. wide surgical excision.
B. Simple mastectomy

The keyword is atypical ductal hyperplasia.

44- A 24-year-old lady presents with a hard, mobile, well-circumscribed painless left
breast mass that has been increasing in size from the past few months and was NOT
related to her menstrual cycle. The most like Diagnosis is?
A. Fat cyst
B. Fibroadenoma
C. Fibrocystic changes
D. Phyllodes Classic case of phyllodes.

45- Around 20 YO patient presented with breast mass that increased in size during the
past year. On palpitation, a mobile mass was noted in the RLQ of right breast, measured
to be around 8*10 cm. Skin thinning around the lesion was noted. What is the diagnosis?
A. Mastitis
B. Phyllode
C. Fibroadenoma
Both Fibroadenoma and Phyllodes continuously grow, but the
difference is that Phyllodes grow rapidly and also cause thinning of
the overlying skin.
46- Female 20 years recently develop mass 2x2 cm that is oval in shape and smooth
wall. what is your Dx?
A - breast cyst
B- fibroadenoma
C - fibrocystic cancer
D - intraductal carcinoma

An oval shaped mobile mass. A missing keyword is the relation


between the mass and the menstrual cycle, unlike Phyllodes.
There’s another recall with multiple oval masses which is again,
a case of fibroadenoma.
Another recall:
21 year old female presented to your clinic with a 2 month history of right breast lump. It
started as 2 cm in size oval shaped and mobile lump. The lump size fluctuate around
menstruation. What is the diagnosis?
A. Breast cyst
B. Normal breast tissue in young people
C. Fibroadenoma
D. Phyllode

47- A 46-year-old female presents with a painful mass 1x2 cm in the upper outer
quadrant of the left breast. There are areas of ecchymosis laterally on both breasts.
There is skin retraction overlying the left breast mass. What is the most likely
diagnosis?
a) Fat necrosis
b) thrombophlebitis
c) hematoma
d) Intraductal carcinoma
e) sclerosing adenosis
The keyword is skin retraction overlying the breast in
addition to ecchymosis.

48- Multiple small breast masses bilateral get worse prior menses
A. Fibrocystic
B. Fibroademoa

49- Female in 33 w present with multiple follicle around the areola of nipple smooth round
and painless
A. Montgomery’s Follicles
B. breast cyst
C. Lactiferous duct
Confined to the areola.
D. Mondor's disease

50- Female patient with unilateral nipple dryness,


crust and oozing discharge..bilateral breast
ultrasound and mammography are normal.
what is the next step?
A. Follow up US in 6 month.
B. Prescribe steroid ointment.
C. Nipple biopsy
D. Referral to dermatology Case of paget disease. As always, the next step after
imaging is core needle biopsy.
51- 35 year old asymptotic lady is seen for counselling regarding her breast cancer
screening. Her mother was diagnosed with breast cancer at age of 67 y and her sister was
diagnosed with breast cancer at age of 45 y, She had no history of breast biopsies.
What would you recommend for her screening?
1- Annual CBE and Mammogram alternating with breast US
2- Annual CBE and Mammogram alternating with breast MRI
3- Annual CBE and Breast US alternating with MRI until she reach 40 then you start with
annual mammogram
4- Annual CBE and Mammogram

The patient is 35 years old, and her sister developed cancer at 45 years old, so
start mammography 10 years early.

52- 50 year old asymptotic lady is seen for counseling regarding her breast cancer screening.
Her 30 years old sister was recently diagnosed with breast cancer, and her mother diagnosed
with breast cancer at age of 70. She had no history of breast biopsies. What would you
recommend for her screening?
1- Annual CBE and Mammogram alternating with breast US
2- Annual CBE and Mammogram alternating with breast MRI
3- BRCA gene testing for her
4- BRCA gene testing for her sister

Since her sister developed breast cancer quite early, it makes us suspect
BRCA. First, confirm that the affected patient has the gene, then start
annually screening the entire family if the BRCA gene is positive.

53- breast mass behind nipple, on US there is hypoecoich lesion cyst what next ?
A- FNA
B- core biopsy
C- exicional biopsy
D- reassess after

Another recall:
26 year old female presented with a tender breast lump. US was done and showed simple
cyst. She does not have any family history for breast cancer. What is the best management?
A. Aspirate
B. MRI
C. Excision
We aspirate breast masses only in a case of cyst.
D. Antibiotics

54- Women in her 50’s with hard non tender immobile breast lump with tethering and red
skin, dx?
Fibroadenoma
Duct ectasia
Carcinoma of breast
Breast cyst Skin tethering? This is cancer.

55- 32y female present with 4cm fibroadenoma with hyperplasia and atypia
What is the most significant risk factor for breast cancer?
Her age
Presence of hyperplasia
Presence of atypia
Fibroadrnoma size
56- Lady with fibroadenoma underwent excision Histopathology result: fibroadenoma +
invasive ductal carcinoma What is the management?
A— Radiotherapy
B— Chemotherapy
C— Mastectomy
D— High risk screening protocol
Both DCIS and invasive ductal are treated with surgery. However, if it were
a case of LCIS, go with D.

57- Breast feeding mother with red swelling in the exam there redness tenderness but no
Fluctuation What is most appropriate management?
A-flucloxacillin
B-Aspiration
C-I&D
Erythema, swelling, no other skin changes, and no fluctuation. Clear case
of mastitis.

58- breastfeeding woman presents with a localized mass on the right upper quadrant of her
right
breast with swollen axillary lymph nodes. What is the most likely diagnosis?
A. Breast abscess
B. Mastitis
C. Breast cancer
D. Duct ectasia Presence of mass and swollen axillary LN.

59- Female postpartum presented with breast pain on examination there was local
erythema, tenderness and thinning of skin how will you manage?
A observation
B incision and drainage
C antibiotics

Due to the thinning of skin. In breast abscess, any skin changes aside from
redness requires I&D.

60- Lactating women present with right breast pain for 6 day. On examination , hot tender
swelling lateral to the right areola. Pt started to take floxacillin Us/ Cystic lesion,
thickened content , ddx could be galactocele, abscess, complicatedcyst for correlation.
What next
1. incision and drainage
2. repeated aspiration
3. Exisional biopsy

I had this question in my exam. In the full recall, the cyst was 3*4cm and
there were no skin changes that warranted I&D.

61- Lady with treated breast carcinoma. Didn’t mention treatment given to her. When can she
get pregnant?
A. 2 years.
B. 9 months.
C. 3 months.
D. 5 years.
Trauma

Chest trauma
Tension Pneumothorax
Hyperresonance on percussion and decreased breath sounds on the same
side, tracheal shifting to the opposite side. Raised JVP. Treated by needle
decompression.
Open Pneumothorax
Suspect it if you notice a suction sound from a lacerated wound, then go for
three side dressing. If not in the options? Go for a chest tube.

Cardiac tamponade
Raised JVP, low BP (weak thread pulse), muffled heart sounds. Bilateral clear
lungs. Treated by pericardiocentesis.

Hemothorax
Stony dull on percussion due to fluid, flat JVP (because of hypovolemia).
Treated by inserting a chest tube.

Flail chest
Multiple broken ribs and paradoxical breathing. No additional signs. Treated by
analgesia + assisted ventilation.

Cardiac contusion
Signs of ecchymosis on the chest, bounding pulse, arrhythmia.

Pulmonary contusion
New lung infiltrates post MVA.

Tracheobronchial Injury
Similar to tension pneumothorax, but has signs of subcutaneous emphysema
and pneumomediastinum. Diagnosed by a bronchoscopy.

Tube placement
Chest tube -> between the 4th and 5th ICS midaxillary line
Therapeutic needle -> at the 2nd ICS midclavicular line
Diagnostic needle -> between the 8th and 10th ICS midaxillary line

Thoracotomy indication
1500ml after chest tube insertion, or 200-250cc within 2-4h
Abdominal trauma
STAB
Stable -> CT
Unstable -> Laparotomy

BLUNT
Stable -> CT
Unstable -> FAST

Approach to management after FAST as an initial step:

Stable and positive -> CT


Stable and negative -> Routine examination
Unstable and positive -> Laparotomy
Unstable and negative -> DPL

Immediate laparotomy:
Positive CT findings, Positive FAST if unstable, omentum is seen, evisceration,
peritonitis, or abdominal rigidity.

Neck trauma
Zone 1 -> CT-A if +ve -> Endovascular repair
If +ve bronchogram or esophagram -> open repair

Zone 2 and 3 if asymptomatic -> observe

Zone 2 symptomatic -> immediate open repair


Zone 3 symptomatic -> CT-A if +ve -> endovascular repair

Unstable (expanding hematoma, uncontrolled hemorrhage) = ligation in all zones


62- 32 years old MVA. prominent neck veins and marked decreased breath sound on right
side of the chest.
A-tension pneumothorax
B- hemothorax
C- cardiac temponade
Full recall. However, it’s enough to diagnose tension pneumothorax. Cardiac tamponade
requires the beck’s triad: 1) low BP 2) muffled heart 3) raised JVP. As for hemothorax,
suspect it if you find stony dullness on percussion and flat JVP. In our question, the JVP is
distended. Tension pneumothorax is the most common presentation and all the details in
the question lead us to it.

63- 25 year old female involved in a snowmobile accident. She is currently dyspneic with
respiratory rate of 40 breath per minute. Breath sound are markedly diminished on the right
side And raised JVP. Which of the following is the first step in the management.
A. Chest tube insertion
B. Pericardiocentesis
C. Needle decompression
D. Chest x-ray

Clear case of tension pneumothorax.

64- Trauma to axilla and lateral chest wall, fracture 4&5 ribs , while examination , suction
sound was sourced from lacerated wound on fractured ribs (no vital data) , next:
A. Chest tube
B. Intubation
C. Urgent Thoracotomy
D. Dressing three side

Open pneumothorax. Initiate management with a three side dressing. Definitive


treatment requires closure of the chest wall defect and a chest tube away from
the site of injury.

65- Patient had a stab wound in the chest. Came with weak thread pulses, raised JVP, Equal
Bilateral Air
Entry in both lungs. What's the diagnosis?
A. Pneumothorax
B. Cardiac tamponade
C. Pulmonary contusion

The beck’s triad in this case is missing muffled heart sounds, but by exclusion
tamponade is still the right answer. Equal bilateral lung sounds excludes
pneumothorax which is unilateral. Weak thread pulse excludes cardiac contusion
because it presents with a bounding pulse in addition to arrhythmia and ecchymosis.
Side note: Weak thread pulse = low BP.

66- Chest gun wound entry was lateral to left nipple and exit point below left scapula, patint
suddenly become worse w/ raised JVP , on auscultation Normal air entry and muffled heart
sound, most appropriate management?
A-fluid bolus
B-Pericardiocentesis
C-Immediate thoratom
The beck’s triad is present here.
67- Patient sustained a chest stab wound injury, on examination there was Stony
dullness over right chest and tracheal deviation to the left, what is the diagnosis?
A. Open pneumothorax
B. Tension pneumothorax
C. Massive Hemothorax

Stony dull immediately excludes tension pneumothorax, as that one presents with
hyperresonance. Hemothorax is treated with a chest tube.

68- A patient with Anterior chest trauma with bruising in the sternum. Patient vitally stable,
clear cardiac and respiratory exam, except for pounding pulse. ECG: Arrhythmia X-ray:
Sternal Fracture. Echo: Normal What is the diagnosis:
A. Pneumothorax
B. Cardiac contusion
C. Cardiac Tamponade
D. Ventricular rupture

Strong bounding pulse, arrhythmia on ECG, and chest ecchymosis.

69- 61 years old women sustain MVA to here left chest brought to ER vitally stable no open
wound admitted for observation, couple of hours later developed suddenly SOB only, initial
CXR reveled 3-7 left ribs features. Repeated CXR showed well demarcated left lung infiltarate.
What is the diagnosis?
A- PE
B- flial chest
C- cardiac contusion
D- pulmonary contusions

The keyword is lung infiltrates.

70- Patient with blunt chest injury resulted in fracture in 3rd, 4th and 5th ribs in more than
one site. What is the initial treatment?
A. Intubation
B. Assisted ventilation
C. IV fluid.

Clear case of flail chest. A missing keyword which is most likely in the full
recall is paradoxical breathing. Start management with analgesia, then go for
assisted ventilation.

71- 35y.o male brought to ER after road traffic accident , complaining of right side chest pain,
he is conscious, alert & oriented. Surgical emphysema in upper chest and neck. Chest X- ray
shows Rt sided pneumothorax &
pneumomediastinum. What is the diagnosis
A- Tension pneumothorax
B- Open pneumothorax
C- Tracheobronchial injury
D- Hemopneumothorax

Subcutaneous emphysema and pneumomediastinum.


72- A 50 year old sustained blunt trauma to the chest with persistent pneumothorax
and significant air leak through double intercostal tubes most appropriate next
procedure ?
A. Reposition of ICT
B. Prompt thoracotomy
C. Endotracheal intubation
D. Fiberoptic bronchoscopy

Significant pneumomediastinum. Tracheobronchial injury is diagnosed by


bronchoscopy.

73- Trauma patient with hypotension, X-Ray reveled: Trachea shifted to the right, expanded
lungs and
widened mediastinum. What is the diagnosis?
A. Massive hemothorax
B. Pneumothorax
C. Thoracic aorta rupture
D. Spontaneous pneumothorax
Mediastinal widening and expanded lungs in addition to tracheal shifting commonly
present in a case of aortic injury. Side note: there’s another recall with ruptured
esophagus instead of aortic rupture in addition to tracheal shifting to the left, but I
believe it is a bad recall as the symptoms are not relevant.

74- Post RTA patient complex femoral fracture and tension pneumothorax, chest tube
inserted and transporting to higher center, in transit the patient is desatting and tachypnoec-
cardic what to do?
A. Intubation
B. Nothing
C. Check for bleeding form fracture site
D. Confirm placement of chest tube

Side note:
Chest tube -> between the 4th and 5th ICS midaxillary line
Therapeutic needle -> at the 2nd ICS midclavicular line Diagnostic
needle -> between the 8th and 10th ICS midaxillary line 1500ml or
200cc within 2-4 hours = thoracotomy.

75- Trauma case (about pleural effusion on?) chest tube inserted. After 15 min there was blood
in the water under seal the amount was 1500 mL, how to manage?
A-Thoracentesis
B-Tube thoracostomy (chest tube)
C-thoracotomy

76- Adult post RTA, was brought the ER in a tertiary hospital, patient is alert and
conscious. CT done which showed: Injury to the thoracic aorta and splenic laceration with
free fluid in the abdomen. (Not perisplenic).
BP: 90/67.
HR: 45 bpm.
What’s the next step? Laparotomy is always done before thoracotomy. And
A- urgent laparotomy. now, we start with abdominal trauma below.
B- urgent thoracotomy.
C- refer the patient to a hospital with vascular surgeon
77- Patient presents with stab wound to the abdomen. After wound exploration, you found
anterior abdominal fascia penetration. His vitals were stable. What is your next step?
A- CT abdomen
B- MRI abdomen
C- Exploratory laparotomy
D- Diagnostic laparoscopy

Abdominal fascia penetration does not warrant laparotomy.

78- Young guy is stabbed in anterior abdomen in a fight after a football match. He presented to
emergency with a 1 cm laceration that is 3 cm above umbilical. He has no pain and vitally
stable. What is the best management?
A. DPL
B. Wound exploration
C. Laparotomy
D. CT scan

Wound exploration is not correct. Go with CT and follow the ATLS.

79- 5 years old with stab wound in lower chest, has abdominal distension
Fast shows free fluid in abdomen
All vitals normal except O2 90
What is the most appropriate management:
Thoracotomy
Tube thoracostomy
Expl lapratomy
Angioembolization

Positive FAST and the patient is stable, so the next step should be CT as we have no
urgent need to rush to the OR just yet. By exclusion only, laparotomy.

80- Patient of stab wound measuring 2 cm penetrating injury with minimal bleeding and
partially omentum exposed, patient is vitally stable,
CT report negative findings,
Next step management:
A. Observation
B. Close the wound
C. Leave the wound open
D. Laparotomy

Omentum was seen, so go for laparotomy even if the patient is hemodynamically stable.

81- MVA came and resuscitation was done after that the patient deteriorated BP 90/60
what to do next?
A. Fast
B. Ex lap

Unstable even after resuscitation.


82- Patient after Motor vehicle accident at ER, vitally stable, on examination showed Lift
hypochondrium tenderness and ecchymosis What is the Most Appropriate test:
A. CT
B. FAST
C. DPL
D. Laparotomy

83- Blunt trauma, stable patient, FAST showed intraperotineal fluid. Next step?
A. CT
B. Ex lap

Since we already did FAST and the patient is stable go for CT scan. If the CT is
positive go for laparotomy regardless of vital signs.

84- 55 year old involved in MVA and sustained a blunt trauma to his abdomen. He his
hypotensive 90 / 63 and HR 104. He is not responding to fluid. FAST is negative for fluid in
pericardium, chest or abdomen.
What is the next best step ?
A. Laparotomy
B. CT scan
C. DPL
D. FAST

Blunt trauma + unstable so we did FAST first, but since it’s negative we go for DPL
this time.

85- Post RTA, abdomen mild tenderness all over, conscious


oriented, Not rigid or severely tender abdomen
Fast: positive for moderate fluid collection
Bp: 90/50
He received 2L crystalloid without improvement
What is next:
-Lavage
-Exploratory laparotomy
-Ct scan

Positive FAST and the patient is unstable.

86- A 44 year old lady was hit by a vehicle, and brought to the emergency room conscious, on
100% 02, received 2 liters of normal saline and 2 liters of blood. Blood pressure 60/40 mmHg
Examination confirmed abdominal rigidity. 145 beat /Heart rate min CXR and pelvic x-ray were
normal. Which of the following is the most appropriate step?
A) DPL
B ) FAST
C) CT scan abdomen
D) surgical exploration

Blunt + unstable so normally we would choose FAST. However, due to abdominal


rigidity we need to go straight for laparotomy.
87- Patient after RTA with sever lower back pain, what to do until the surgeon comes:
A. CT whole spine
B. Flextion extension test
C. Restriction of spine movement
D. Pelvic binder

A similar recall: How to exclude cervical injury? Cervical CT scan.

88- A female get high energy accident (high velocity) with seat belt sign , On X- ray
has chance fracture . What will associated with this fracture?
A) Duodenal perforation
B) Gastric perforation
C) Jejunum perforation
D) Vena cava perforation

Seatbelt + Chance fracture.

89- A 25 years old man was brought to the ER after being involved in a motor vehicle
accident. He opened his eyes spontaneously and responded appropriately to verbal
commands. His respiration wasz shallow and he had a left chest wall contusion. He was able
to shrug his shoulders but unable to move his elbows or lower limbs. BP 80/40, HR 70, RR 30.
Which of the following is the most likely cause of hypotension?
A. Cardiac tamponade.
B. Internal hemorrhage.
C. High spinal cord injury.
D. Tension pneumothorax. Low BP + low HR. Neurogenic shock.

90- Patient underwent surgery after abdominal gunshot splenectomy pancreactomy removal
of parts of intestine and did Hartman procedure. Second day he deteriorate what the most
appropriate next step?
A-Exploration
B-US
C-X-RAY
D-CT
Unclear question, but if the patient deteriorated it’s safe to assume laparotomy is a
safe answer.

91- Patient with penetrated neck in the zone 3 and he is having active bleeding , CTA report
shows : Avascular injury
what is the best management?
A. Open and primary repair
B. Endovascular
C. Open and ligation

92- Neck penetrating injury on zone 1 with emphysema, next step?


A. Neck exploration
B.CT head and neck
Then go for angioembolization if positive.
C. Angioembolization
Appendicitis
Appendicitis is the main cause of an acute surgical abdomen. The most
common SMLE question is always about investigation. It depends on the age
group, presentation length, and the Alvarado score.

Pediatric and female (reproductive age) -> US to avoid unnecessary radiation


for children and a potentially pregnant woman. Order B-HCG to exclude
pregnancy if it’s available in the choices, if not go with US.

Elderly (suspect cancer) and atypical lengthy presentation (suspect mass/


abscess/perforation) -> CT scan

In a typical male presentation, go with CT if the Alvarado score is 1-6. Go


with laparoscopy if it’s 7-10.

Alvarado score
3 Symptoms -> Anorexia, N/V, shifting of pain to RLQ.
3 Signs -> RLQ tenderness, rebound tenderness, fever.
2 Labs -> Leukocytosis, left shift (neutrophils).

Each of them add a single point, except for RLQ tenderness and
Leukocytosis, they add 2 points.

Signs
Rovsing’s = LLQ palpation causing RLQ pain
Bluberg’s = rebound tenderness
Psoas = retro-cecal inflamed appendix
Obturator = pelvic inflamed appendix

How to find the appendix during the surgery? Follow the taeniae coli.

If you go for an appendectomy and during the procedure you discover a


normal appendix go ahead and search for Meckel's diverticulum. Afterwards,
what we do next depends:
If open -> proceed with appendectomy
If lap -> leave the appendix

if you find an inflammation in the terminal ileum -> refer the patient to
gastroenterology as it’s a case of IBD.
Appendiceal mass

Percutaneous drainage of large collections then schedule for an


interval appendectomy in 6 weeks. Choose conservative if the
collection is small.
Go for colonoscopy in an old age patient.

Appendiceal cancer

In adenocarcinoma, always go for a right hemicolectomy.

In carcinoid it depends:

<2cm / tip of the appendix / no mesothelium invasion / no lymph


node involvement -> Observe

>2cm / base of the appendix / mesothelium invasion / lymph node


involvement -> Right hemicolectomy
93- 9 year old came with an 8 hours history of abdominal pain and nausea.
What’s the most common surgical emergency:
A- Appendicitis.
B- intussusception.
C- cholecystitis.
D- pancreatitis.

94- Child with clear case of appendicitis, most appropriate investigation to reach diagnosis?
CT
US
US is enough. No need to cause unnecessary radiation.

95- 25 y.o female, married, with 12h of RLQ pain, N/V. Vitally stable. No labs provided. Next?
A. CT
B. US
C. Diagnostic laparoscopy
D. Exploratory lap.

It could be ectopic, so with these choices go for an US next as it may be enough. Ideally,
we would order a pregnancy test first then go for CT if the Alvarado score is less than 7
and the ectopic pregnancy has been excluded.

96- Young female came to ER c/o RIF pain for 12 hours, on PE: there is tenderness in
Suprapupic + RIF , and no rebound tenderness,
WBC high 14k
On US : inconclusive
What is the next step ?
A- CT
B- open appendectomy
C- diagnostic laparoscopy
D- Transvaginal US
In this question, they already did an US and it was useless. Again, the correct answer should be
pregnancy test. It seems to be a typical case of appendicitis but why rush to CT and cause
potential harm if the patient turned out to be pregnant? Between these two options, the most
likely answer is transvaginal US since not only is it the preferred way to diagnose an ectopic
pregnancy, but it would also diagnose ovarian cysts. Lastly, the question specified “what is the
next step” so in my opinion we should start with it first to rule out obgyn causes.

97- 27- year-old obese woman presents with right iliac fossa pain associated with anorexia,
nausea, and vomiting. On examination, there is moderate right iliac fossa tenderness. Labs
showed: leukocytosis, what is the most appropriate management?

A. Open surgery
B. CT
C. US
D. Diagnostic laparoscopy

US has a limited role in obese patients. Open surgery makes no sense. Between B and D, I
would go with CT scan as the “most appropriate management” once we order a pregnancy
test first since the Alvarado score in this case is 6. As explained previously, the most
appropriate management is not the same as the “most appropriate next step” so the
answer here is CT scan.
98- 17 years with history of right iliac fossa pain rebound tenderness +ve guarding. what is
the investigation next :
A- laparoscopy
B- US
C- CT scan
In the full recall, this patient had an Alvarado score of 7, so we should go for laparoscopy. If
less than that the answer changes to CT.

99- 24-year-old male comes with RLQ pain, positive rebound tenderness after 2 hours of
playing
football. His WBC count 6 (normal), INR 1.1 (normal). Which of the following is the most
appropriate next investigation?
A - CT abdomen
B - No need for further investigation, and follow up after 1 week
C - US or Xray

100- Case of pregnant 16 weeks developed right side abdominal pain (did not specify upper
or lower). Temp 73.8
Most common cause
A) gastritis
B) pancreatitis
C) cholecystitis
D) appendicitis

They did not specify the location, but the most common cause is appendicitis.

101- Female 16 week pregnant, came with right sided abdominal pain (RUQ) associated with
nausea and vomiting. The doctor ruled out all pregnancy related abdominal pain. What's the
most likely diagnosis?
A- Gastritis
B- Pancreatitis
C- Cholecystitis
D- Appendicitis

Here, they specified RUQ which often means a case of cholecystitis. However, keep in mind
that the appendix changes its location upwards in the third trimester. According to the recall,
our patient is in the second trimester but it could be a bad recall and the patient is actually in
the 26th week. The answer would change to appendicitis in that case.

102- Patient with appendicitis, when palpating the RLQ it causes pain in the LLQ. name of this
Sign?
A- Psoas
B- Rovsing
C- Obturator
D- Blumberg

Bad recall. Palpating the LLQ causes RLQ pain.


103- Pt with appendicitis, during surgery the surgeon didn’t find the appendix
What is the most effective way to locate the appendix?
A. Palpate ileocecal valvd
B. Follow terminal ileum
C. Follow the confluence of the tenia coli
D. Ileocecal valve

If C isn't in the choices? Go with ileocecal valve.

104- During Lap cholecystectomy you found AAA 4.5 cm what you will do?
A- Do the surgery and FU with US
B- Do the surgery and follow up with CTA
C-Do CTA before the surgery
In the full recall, the patient was also asymptomatic. We follow
asymptomatic known cases of AAA with an US. If otherwise, CT-A
105- Man diagnosed with appendicitis due to rigidity and abdominal tenderness
During the surgery the appendix was fine, what to do now?
A- Remove the appendix and close
B- Look for meckles diverticulum
C- Do open surgery and explore
D- Close without doing anything

Missing information. Was it laparoscopic or open? If you go for an appendectomy and during
the procedure you discover a normal appendix go ahead and search for Meckel's diverticulum.
Afterwards, what we do next depends:
If open -> proceed with appendectomy
If lap -> leave the appendix
If no pathology has been found? Go for appendectomy
regardless.
106- 35 YO male presented with typical picture of acute appendicitis. He was taken to OR for
laparoscopic appendectomy. During OR the appendix appears normal with no inflammation.
Both cecum and terminal ileum appears inflamed. What is the best action?
A. Right hemicolectomy
B. Close without further intervention
C. Appendectomy and refer the patient to gastroenterology
D. Take biopsy of cecum and terminal ileum

Refer the patient to gastroenterology as it’s a case of IBD. The other choices don't make
sense.

107- 21 year old patient with right iliac fossa pain for 7 days on examination: palpable mass at
the iliac fossa
WBC high
US done showing appendicieal mass with NO collection
What to do:
A) non interventional
B) lap appendectomy
C) open appendectomy
D) exp laparotomy

No collection. Conservative care followed by an interval laparoscopic appendectomy.


108- Young year old man with appendicitis was treated conservatively with antibiotics. He
now presents with an appendicular mass with no collection. How will you manage this case?
A. Interval laparoscopic appendectomy after 12 weeks
B. Interval open appendectomy after 12 weeks
C. No further intervention needed
D. Colonoscopy after 6 weeks

Same approach we took with the prior patient.

109- Patient presented with symptoms of appendicitis.


Imaging: appendicitis, fecolith, appendicular abscess 10 x 15 cm reaching the flank.
What is the management?
A- Open drainage
B- Percutaneous drainage
C- Open appendectomy with drainage
D- Laparoscopic appendectomy with drainage

Large collection this time. Drainage then interval laparoscopic appendectomy.

110- 58 year old man with appendicitis was treated conservatively with antibiotics. He now
presents with an appendicular mass with no collection. How will you manage this case?
A. Interval laparoscopic appendectomy after 12 weeks
B. Interval open appendectomy after 12 weeks
C. No further intervention needed
D. Colonoscopy after 6 weeks

Old age? Colonoscopy.

111- Pathophysiology regarding patient with appendicitis complicated by Appendicular mass


(Abscess), patient was Feverish (38.7)?
A. Peripheral Vasoconstriction
B. Decreased Cardiac Index
C. Redistribution of Blood
D. Bradycardia

112- 26 year-old male patient underwent appendectomy after signs and symptoms of
appendicitis. Histopathology report: 1 cm carcinoid at tip of the appendix . What is the
appropriate management?
A. Observation.
B. Chemotherapy
C. Radiotherapy
D. Right hemicolectomy

113- 26 year-old male patient underwent appendectomy after signs and symptoms of
appendicitis. Histopathology report: 2 cm carcinoid at Basie of the appendix . What is the
appropriate management?
A. Observation.
B. Chemotherapy
C. Radiotherapy
D. Right hemicolectomy

Base of the appendix. Review the approach outlined at the start of this section.
Hernia
Small bowel obstruction is mainly mechanical, and hernias are the most
common cause for SBO in a non-operated abdomen. Adhesions is the most
common cause if prior surgery exists. Paralytic illeus is another cause for
SBO occurring after surgery and features a silent sluggish bowel on
examination with no mechanical blockage, so you’ll need to check for
electrolytes first. Nitrous oxide is C/I in a SBO case.

Almost all adult hernia cases are repaired through hernioplasty. Open hernia
repair is preferred over laparoscopic, except in the following cases:
- Bilateral hernia
- Recurrent hernia
- Obese

If the previous repair was open go for laparoscopic, and the opposite is true.

Para-umbilical hernia (PUH)


If below 2cm go with suturing. Larger? Mesh repair.

An asymptomatic or minimally symptomatic reducible inguinal hernia in the


co-morbid elderly patient doesn’t need repair as the risk of strangulation is
low. Same scenario but irreducible? Go for urgent repair. What if it was
femoral hernia instead? Always needs repair regardless.

Post operative
Pus oozing? Open the wound to let it drain. If it reaches the mesh? Remove
the mesh as well.
Swelling only? This is a collection of serous fluid called seroma, which
simply needs regular wound dressing.

Complications
Incarcerated -> irreducible only.
Obstructed -> mainly presents with SBO symptoms and air fluid level.
Strangulated -> mainly ischemia and necrosis causing skin color changes. In
addition to potential SBO sx, the skin overlying the hernia is erythematous
and tender on examination.

Urgent surgical exploration is indicated in any of these cases.

Pediatrics
Umbilical hernia -> observe until 5 years old, then go for herniotomy.
Inguinal hernia in a child? Always go for herniotomy.

Minimal time before lifting heavy weights -> 6 months


Ideal time before lifting heavy weights -> 1 year
114- 24 years old male medically and surgically free, presented with manifestation of intestinal
obstruction, suspected to be small bowl obstruction, what is the most common cause?
A. Hernia
B. Adhesion
C. Meckel’s diverticulum
D. Malignancy

115- A patient underwent a hernia repair operation 12 years ago, now presents with signs of
intestinal
obstruction. Which of the following is the most likely cause?
A. Improper Repair
B. Late Onset Crohn's
C. Adhesions
D. Cause Is Irrelevant to The Prior Surgery History

116- Pt after splenectomy surgery presented with with multiple episodes of intestinal
obstruction Now complianing of abdominal destination pain rigidity sluggish bowel sounds
Vitality stable
X ray air fluid level
What is the intial management?
A- Paracentesis
B- gastrografin enema
C- expl laparotomy
D- NGT , analgesic, bowel rest

Most likely to be adhesions due to the intestinal obstruction + hx of surgery and air
fluid levels. A rigid abdomen is an indication for laparotomy but the patient is vitally stable
so I think it’s a bad recall. Either way, if it truly came like this I would still go with
D since the patient is stable and the question asked for an initial management.

117- ER. Abdominal pain generlized tenderness hx of open laparotomy 10 years ago
abdomen distended imaging multiple air levels distended small intestine fever no vitals what
is the best?
A. Exp lap
B. Diagnostic laparoscope
C. Abx
D. CT

Adhesions again. The best way to diagnose intestinal obstruction in general is through a CT
scan.

118- A 57 year old man presents with 2 days of abdominal pain, nausea and vomiting.
Examination revealed a distended, tympanic and mildly tender abdomen without rebound or
guarding (see lab results and reports) Hb 153 (normal value 130-170 g/L)
WBC 12.6 (high) Plain abdominal x-ray: multiple air-fluid levels CT scan: multiple dilated
loops of small bowel with a transition point in the distal small bowel, with some adjacent fat
stranding. Which of the following is the most appropriate next step in management?
1.Observation
2. Colonoscopy
3. Diagnostic laparoscopy
4. Exploratory laparotomy
They have already done CT here. Our patient is stable so the next step is to observe and keep
NPO. No role for colonoscopy and no need to jump straight to laparoscopy.

119- 24 years old with history of appendectomy 5 years ago present with abdominal pain,
distintion , vomitting for 3 days Ct scan show obstruction signs And peritonitis , Whats
contraindicated in this patient
A. Propofol
B. Ketamine
C. Sevoflorane
D. Nitros Oxide
120- 5 days post open appendectomy, signs of bowel obstruction, abd distention, bowel
silent, (no vitals) most appropriate management
A. chech electrolyte
B. Rectal tube
C. CT ?
D. laparotomy

A case of paralytic illeus. It is linked to electrolytes imbalance.

121- Female post op with paralytic ileus. Labs showed hypokalemia.


What is the best investigation?
A - Urine osmolarity
B - Urine K
C - Stool K
D - ECG

Hypokalemia? Go for an ECG because of the effect it has on the heart.

122- Elderly female, presented to ER with symptoms of bowel obstruction and negative past
surgical history. She is stable with distended soft and lax abdomen. What is the next step in
management?
A. Chest Xray
B. Abdomen Xray
C. CT abdomen
D. Groin examination

Since it is such a common cause of SBO, we have to rule out inguinal hernia first with a
simple physical exam.

123- A boy was playing football and felt sudden pain in his groin while kicking the ball or
aiming to the goal smth like that, there is a swelling at the groin but no cough impulse and not
reducible, vitals showed fever and wbcs were high, whats ur management?
A. aspiration to rule out hematoma
B. give analgesics and check after 8 hrs
C. surgical exploration for hernia
D. force reduction manually or smth like that

124- 45 years old patient, came with history reducible hernia in inguinal area it pop out every
two days with mild pain and he can manuel reducible it by his finger, then he suddenly
developed severe constant pain in early morning in that area and came to hospital, on
examination of the inguinal area and scrotum examination and he is free from both them and
pain is disappear, Now just have persistent nausea and vomiting, what is the next step ?
A. Pelvic and Abdomen CT
B. Hernia repair today after 2-hour
C. Reassurance and discharger
D. Hernia repair after 2 days

Since the patient is not complaining of pain at the moment there is no urgent need to head to
the OR just yet, so we might as well perform a CT scan and evaluate further. What if he was
still in severe pain? Urgent surgery, not in 2 hours.
125- 76 years old male with reducible inguinal hernia and moderate pain, what’s the treatment:
A. Open mesh repair
B. Non-surgical procedure
C. Simple repair

In cases of inguinal hernia affecting the elderly, the decision to go for repair depends on how
the severity of symptoms. Since the patient is minimally symptomatic he should be treated
conservatively since the risk of complications is low.

126- 75 years old male, came with inguinal hernia, iireducible and painful mildly, afebrile, no
signs of inflamation :
A- Open repair with mesh
B- Repeai hernia
C- do nothing

This is a similar case, but the hernia here is irreducible, so this patient needs surgery. We
prefer an open repair over laparoscopic except in the following cases: bilateral, recurrent (open
repair hx), and an obese patient. An irreducible hernia implies one of a few possible
complications: incarcerated, strangulated, or obstructed. All three need surgery regardless.

127- 65 years old female, with Asymptomatic femoral hernia. What is the most appropriate
management?
A. Observation
B. Open repair with mesh
C. Laparoscopic repair

The same could not be said for femoral hernias. They always need repair because of the high
risk of complications.

128- Adult soldier with bilateral inguinal swelling with positive cough impulse what is the most
appropriate management?
A. Laparoscopic repair
B. Herniotomy
C. Open hernia repair and mesh
D. Observation

Since it’s bilateral.

129- 30 years old male had open hernia repair few years ago, presenting now with reducible
inguinal hernia on
one side extending to hemiscrotum. what is the appropriate thing to now?
A. Laparoscopic repair with mesh
B. Open repair
C. CT abdomen
D. US

Recurrent hernia.
130- 20yo male patient came to ER complaing from abdominal pain.
O/E there was inguinal hernia that was tender and irreducible.
On x-ray there was multiple air fluid level in abdomen.
What type of hernia?
A. Strangulated
B. Obstructed
C. Irreducible
D. Incarcerated

Let us analyze this one by one in an ascending manner.


Incarcerated and irreducible are the same thing, meaning they’re both excluded. Their
only feature is an irreducible hernia, which often presents with the other types
anyway. That leaves us with strangulated and obstructed. Obstructed may be
irreducible and it mainly presents with SBO symptoms and an air fluid level.
Strangulated hernia may present with all of the above in addition to a tender hernia on
examination and ischemic skin color changes. In other words, red skin. The keyword
in this question is tender hernia, so the answer is strangulated.

131- Post laparoscopic hernia repair 7 days ago, came now because of pus coming out from the
wound, you found 2*3 cm pus, and it is oozing from the wound, no fever in scenario,
managment?
A. Abx
B. Drainage
C. Drainage remove the mesh
D. Open the wound and leave it open
Simply open the wound to let it drain. What if the pus reached the mesh? The answer would
change to C. Another recall mentions that the mesh is “exposed” meaning it reached it.

132- Child 4 years old with asymptomatic umbilical hernia what's the treatment?
A. Reassurance
B. wait until 5 years
C. Surgical repair
Umbilical hernia in children depends on the age. Below 5 years old? Observe. Once the patient
turns 5 we go for herniotomy.

133- Newborn with right reducible inguinal Hernia , high riding testis ?
Most appropriate management?
-Right inguinal herniolomy
-Open repair
-Mesh repair
-Wait till 6 Yo

Inguinal hernia in children needs herniotomy now.

134- man did an operation for a ventral hernia. What will you tell him?
a. Use laxatives or stool softeners for 6 months
b. Do not lift heavy objects for 6 months
c. Wear a hernia belt for 3 month

The minimum time before lifting weights is 6 months, but ideally we should wait for 1
year. It depends on what the question is looking for.
Gastrointestinal
Upper GI
Boerhaave syndrome (esophageal perforation) is characterized by rupture
of the esophagus due to forceful vomiting.

Esophageal cancer
The most common type worldwide is SCC, the most common risk factor
overall is smoking, and the most significant risk factor overall is Barret’s
esophagus.
- The strongest risk factors for SCC is achalasia (x10) followed by smoking
(x9). The cancer is located in the proximal-mid esophagus.
- The strongest risk factor for adenocarcinoma is metaplasia due to
Barrett’s (>x10), followed by GERD (x5-7). The cancer is located in the
distal portion of the esophagus.

Upper GI bleeding approach


ABC -> endoscopic dx / mx -> surgical mx if warranted -> chronic
management

Peptic ulcer is the most common cause of upper GI bleeding and it may
only present with bleeding initially. Suspect it in cases with NSAID use.
Urea breath is essential to confirm eradication of H. Pylori as clinical
assessment alone isn’t enough. It’s treated medically unless uncontrolled.

Duodenal ulcers
1) Anterior perforation? Go for a simple closure with an omental patch.
2) Posterior bleeding? You will need to stop the bleeding by ligating the
gastroduodenal artery. Suture the bleeding site + go for truncal vagotomy.

A known case of PUD who failed medical treatment had an endoscopy


showing multiple ulcers in the antrum? Go for a partial distal gastrectomy.
To diagnose gastric cancer if suspected, go for an endoscopy with biopsy.
To stage gastric cancer, go for an endoscopic ultrasound. Surgery is the
definitive treatment.

Low grade mucosa lymphoid tissue (MALT) -> simply eradicate H. Pylori
with medical treatment.

If the patient has repeated episodes of vomiting then starts to bleeds, it


goes with Mallory Weiss which needs upper endoscopy regardless of
patient status to confirm it before discharging the patient.

Erosive esophagitis presents with a metallic taste and retrosternal chest


pain.

Esophageal varices is suspected in patients with signs of portal


hypertension/abnormal LFT.
Lower GI
Lower GI bleeding approach
ABC -> NGT aspiration (if massive) -> colonoscopy -> if normal
colonoscopy, TC99 to look for angiodysplasia

Diverticular disease (a complication of diverticulosis) is the most


common cause of lower GI bleeding.

Diverticulitis presents with LLQ pain. Typically, treatment is


conservative with antibiotics, but drainage is indicated for unstable
patients or a large size collection.

Elderly + recurrent painless bleeding and normal mucosa on


colonoscopy -> Angiodysplasia. Diagnosed with TC99/Nuclear scan.
If we had the same case but for a pediatric age group? Then it’s a
case of Meckel’s diverticulum. Same investigation.

Anal fissure -> Pain with defecation, itchy, and a crack will be seen.
Preventable condition. Treated with lateral internal sphincterotomy.

Anal abscess -> swelling or mass, fever, painful, high WBC.

Prolapsed pile -> painless, vitally stable

Thrombosed pile/hematoma -> painful, vitally stable

Simple low lying fistula -> Pain and discharge. May rupture. Treated
with fistulotomy. The most common location is medial posterior.

Hemorrhoids grades and treatment


1 - no prolapse.
2 - spontaneously reduced, conservative then band ligation
3 - manually reduced, conservative then band ligation if bleeding, and
hemorrhoidectomy if no bleeding.
4 - cannot be reduced. Hemorrhoidectomy.

Old age patient with hemorrhoids? Always go for a colonoscopy first.

Complex fistula approach


Known case of Crohn’s? Go with an MRI. Unknown case? Go with
colonoscopy to confirm Crohn’s as a perianal fistula may be the first
presentation that makes us suspect crohn’s. Surgery for toxic mega
colon is curative in ulcerative colitis, but not in Crohn’s disease.

Complex fistulas are recurrent, multiple, and mainly present with


discharge. The first step after performing an MRI is to administer IV
Abx, then Infliximab for recurrent cases.

Be careful, the Crohn’s case may present with a simple tender


swelling without discharge, which is a case of typical abscess -> I+D
135- 50 years old female presented to ER with sudden left lower chest pain and epigastric
pain, after forceful
vomiting. On examination there was decreased breath sound in the left lower chest What is
the most
likely concern?
A. Pneumonia
B. Pneumothorax
C. Aspiration
D. Esophageal Perforation

Boerhaave syndrome.

136- Post dilatation of esophageal stricture came back with chest pain and
change of voice./
Patient after esophageal balloon dilation for esophageal varices started to complain of
difficulty in breathing and voice change.
What is the complication?
A- Bleeding
B- Perforation
C- Aspiration

Perforation after dilatation can have a couple of different effects depending on where it
happened. Upper part near the larynx leads to voice change, while perforation in the lower
part of the esophagus leads to epigastric pain.

137- case of achalasia. What is the best management choice?


A. Calcium channel Blocker
B. botulism toxin injection at LOS
C. Pnuemonatic dilatation
D. Lower esophageal myotomy

The best is heller myotomy, and the initial choice is pneumatic dilatation.

138- 65 YO female presented with history of progressive solid and liquid dysphagia for 9
months. Associated with vague epigastric abdominal pain after eating. Her labs are all normal
except for mild anemia. He sense somthing in his neck What is the most likely diagnosis?
A. Achalasia
B. Esophageal web
C. Esophageal squamous cell carcinoma
D. PUD

Old age and anemia goes with esophageal cancer. In another recall, the patient is younger and
there is no mention of anemia. Instead, she has an elevated ESR. Therefore, in that recall the
answer is achalasia. Make sure to review the esophageal cancer related questions on most
common/significant as they like to play around with that. Also, in high grade dysplasia SCC
we go for surgery. But if low grade, give the patient PPI and follow up after 6 months.
139- 65 years old women, underwent elective colon resection.
Day 5 post op she complained of abdominal pain & distention.
While preparing her for CT scan, she had fresh blood & coffee ground in her NGT tube bag.
Her vitals & HgB levels are normal.
What is the most appropriate diagnostic test?
A- Colonoscopy
B- CT scan
C- Upper GI endoscopy
D- Abdomen US
As discussed in the upper GI approach, once we stabilize the patient we should prep for an
endoscopy.

140- 70 years old male on Mechanical ventilation (MV) admitted to the ICU due to intracranial
hemorrhage, 7
days later he developed ground coffee vomitus. What is the diagnosis?
A. Stress gastritis
B. Helicobacter Pylori gastritis
C. Dyspepsia
The keyword is mechanical ventilation which led to sores.

141- Post colonic surgery, a patient develops abdominal pain and distention. He is vitally
stable. There is fresh blood and coffee ground aspirate from the NG tube.
What is the diagnostic modality of choice?
A - Upper GI endoscopy
B - Abdominal CT

142- Patient who is on NSAIDS for joint pain, presented with sudden severe continuous
abdominal pain. He
denied any history of vomiting, constipation, diarrhea, and or hematemesis. The pain is
located in the epigastric area with a feeling of Nausea. Upon examination. There is tenderness
and rigidity, what is the
diagnosis?
A. Esophagitis
B. Acute gastritis
C. Perforated peptic ulcer
D. Boerhaave syndrome
History of NSAID use is classic for PUD.

143- Male patient in his 20s, present with sever epigastric pain for 6 hours, start suddenly. On
examination there is diffuse tenderness, sluggish bowel sound. Labs: high WBCs, Amylase
300, Which of the following best initial diagnostic step?
A- Erect chest X-ray
B- Abdominal X-ray
C- Abdominal CT
D- Abdominal US
Perforated PUD. Order an erect x-ray to look for air under the diaphragm.
144- Male patient complains of episode of hematemesis. normal past medical
history Labs show mild anemia, all labs are normal except elevated urea level in blood, what is
the diagnosis?

A. Mallory Weiss syndrome


B. Erosive gastritis
C. Peptic Ulcer Disease

Hematemesis alone goes with PUD, as it’s the most common cause of UGIB.

145- Patient with history of vomiting for 24 hours due to food poisoning. He came today to
emergency department complaining of hematemesis. Nasogastric tube was done and showed
fresh blood.
What is the most likely diagnosis?
A. Gastritis
B. Dieulafoy lesion
C. Peptic ulcer
D. mallory weiss syndrome

Here, the patient had episodes of vomiting before hematemesis occurred.

146- Male pt complain of recurrent heartburn increase when lay down at night, he felt a bitter
taste in his mouth when he was lifting weight at gym, diagnosis?
A. Esophagitis
B- PUD
B. Gastritis
C. Boerhaave syndrome

Metallic taste.

147- 35 years old Adult male medically free presented to ER C/O several episodes of
hematamesis for the first time, takes no medication, no family history of similar attacks, normal
abdominal examintion no guarding no tenderness , endoscopy done and showed dilated
oesophageal bleeding vessels with some adherent clots , Hgb : 9 Plt: 250 Alk: slightly decrease
All other test were normal What is the Diagnosis:
A-Erosive gastrisis
B-PUD
C-mallory weiss syndrome
D-Esophageal varices

The keyword is dilated esophageal bleeding.

148- Elderly patient, came with upper GI bleeding. Hx of PUD, recently completed ttt of H-Pylori.
Endoscopy found arterial duodenal bleeding, failed to be controlled. Received 4 units of pRBCs,
BP 110/60, pulse 12. Best surgical option ?
A. Suture bleeding
B. Suture bleeding site and truncal vagotomy
C. Truncal vagotomy and antrectomy

A case of uncontrolled PUD leading to posterior perforation. The bleeding site is the
gastroduodenal artery which needs to be ligated.
149- Patient have perforated duodenum what to do?
A) partial gastrectomy
B) gastrojejunostomy
C) patch omentum
D) forget

Anterior duodenal perforation treated with an omentum patch.

150- Patient known case of peptic ulcer disease in medications but failed to respond,
upper endoscopy done and revealed multiple ulcers in antrum. What is the best
treatment?
A. Pyloroplasty and vagotomy
B. Total gastrectomy
C. partial distal gastrectomy (anterectomy)

151- patient came to the clinic for a check - up , he has some abdominal pain ,
endoscopy done for him revealed multiple ulcers . The biopsies were taken and
showed positive H.pylori and mucosa - associated lymphoid tissue ( MALT
lymphoma ) . The management should be ?
A. H.pylori eradication
B. Surgical resection
C. Chemotherapy
D. Anti - acid medication

152- 16 YO came to ER after vomiting once with blood , she had recurrent N/V before
her period in the last time there was slight blood with vomiting , after 4-6 hours they
mention also ,all labs and exams are normal what you should so?(came in nov and dec)
A— admit her for observation
B— reassure and ask to come if it recur again
C— prepare for urgent EGD
D— Discharge

A case of Mallory Weiss which requires an upper endoscopy regardless in all suspected
patients. The vomiting was severe enough to cause tears in the esophagus that led to
the blood coming out so why would I reassure and discharge before doing any further
workup?
153- 50 year old male patient presented to ER with massive lower GI bleeding, he is
tachycardic. intravenous crystalloids resuscitation is started and standby PRBC units are
ordered. What is the next step?
A. Colonoscopy
B. NGT aspiration
C. Activate massive transfusion protocol
D. Consider RBC scan

First step in massive lower GI bleeding to rule out UGIB. If not massive, direct colonoscopy.
After NGT, go for a colonoscopy to investigate.

154- pt the fresh blood per rectal, they did NGT and it was greenish ,colonoscopy
report show normal has no colorectal ca , what is next step ?
A. upper GI endoscopy
B. 99 tec

They did both NGT and colonoscopy here. Since the colonoscopy results show nothing of
note, the next step is TC99 to look for angiodysplasia.

155- 3yrs child , the mother noticed blood in his diaper for 2 days , no abdominal pain or
constipation , he has similar episode 3months back .. what is the Dx ?
A- juvenile polyp
B- Michele’s diverticulum
C- Intussusception
D- hirschsprung disease

Similar presentation to angiodysplasia with the only difference being the age group.

156- 65 yrs old male complaining of sever Lower GI BLEEDING


What is most common cause
Angiodysplasia
IBD
Diverticulosis

157- 28 years old patient medically free presented to the ER with hx of abdominal pain
nausea anorexia and lower quadrant abdominal pain vitally stable CT: diverticulitis with 20
ml fluid 2*2 cm What is the most appropriate management
A. Exploratory laparotomy with Sigmoidectomy
B. Exploratory laparotomy with easy content
C. Conservative IV antibiotics
D. Diagnostic laparoscopy

Uncomplicated diverticulitis treated conservatively. Small size, and no air fluid level
indicating perforation.

158- Case of diverticulosis pt came with severe left iliac fossa pain and constipation,
tenderness upon palpation
Bp was normal temp was 37.6
Radiology image
Collection of fluid 9x10 cm with inflammation of the sigmoid
How would you manage
A-Exploratory laparotomy
B-Sigmoidectomy and anastomosis
C-Percutanous drainage

Here, it is complicated diverticulitis which is managed with percutaneous drainage.


159- 4 y.o his went to doctor because parents recognize blood in stool this the only symptoms
Then the doctor examined him and said it’s common and can be preventable:
A- anal fissure
B- hemorrhoids
C- juvenile poly
D- Intusseption

160- 25-year-old man presented to the Emergency Department with severe pain during and
after defecation for 3 days associated with passage of a small amount of fresh blood after
defecation. Physical examination confirmed an acute posterior anal fissure. Digital and
proctoscopic examination were not performed due to the anal pain. Which of the following is
the most appropriate management?
A. examination under anaesthesia
B. lateral internal anal sphincterotomy
C. chemical sphincterotomy with diltiazem
D. botulinus toxin paralysis of anal sphincter

If failed, lateral internal sphincterotomy.

161- Patient with anal fissure not responding to drug, what procedure is most suitable
for his condition?
A. Lateral internal sphincterotomy
B. Anal curettage
C. Lateral External sphincterotomy.
162- About a fistula with discharge, located posterior to anal sphincter what is your
management?
A. fistulogram
B. fistulotomy
C. lateral internal sphincterotomy
D. MRI
163- scenario of anal fistula on 7 o'clock planning for fistulotomy asking about common
are of internal able fistula
A. medial posterior
B. medial anterior
C. lateral
D. supination
164- 28 years old patient came with severe perianal pain and swelling. On examination,
there is 1x1 cm perianal swelling with tenderness. Vitals: normal, no fever. Labs: WBC 8
(normal). Which of the following is the most likely diagnosis ?
A. Anal fistula
B. Anal fissure
C. Perianal abscess
D. Perianal hematoma

Let us exclude one by one.


Fistula -> pain and discharge. May rupture.
Fissure -> pain during and after defecation, itchy, and a crack will be seen.
Abscess -> mass, fever, painful, high WBC.
On the other hand, anal hematoma/thrombosed pile -> pain and swelling, but vitally stable
just like our patient.
What about prolapsed pile? Similar to thrombosed piles, but painless.
165- Patient with rectal bleeding 5 and 7 o clock, sclerotherapy done, what type of
hemorrhoid treated?
A. Internal
B. External
C. Prolapse
D. Thrombosed

166- Pt pain less defication, Blood in th stool. Biles reduced spontanius in 3 areas 3.7...._ Hb
very low, Dx hemmorriid was given which of the following is Definitive management ?
A. conservative manag
B. fiber diet
C. rubber band ligation
D. hemorroidectomv

An example from the management outlined above. Grade 2 (spontaneously reduced) and
bleeding, so the definitive management here is C.

167- 47- year-old female presents with iron deficiency anemia. She has 3rd degree
hemorrhoids. Which of the following is the most appropriate thing to do?
A. Colonoscopy
B. Hemorrhoidectomy 40 years and older in a case of hemorrhoids.
C. CT scan

168- Patient diagnosed with Ulcerative Colitis 6 years ago. when to be screened for colon
cancer?
A. Annually till age of 50
B. If there is mild inflammation
C. At time of diagnosis
D. 8-10 years after diagnosis

169- 25 y/o women taking steroid for IBD C/O abd pain + billous vomiting O/E her abdomen
is distended and tender in the right iliic fossa she had done colonoscopy 2 week ago which
was normal, contrast barium showed : single stricture at terminal ileum, 1 cm from illiocecal
valve. Which of the following is most appropriate management ?
A- strictureplasty
B- right hemicolectomy
C- coservitve management
D- segmental resection with ileostomy

Resection is superior to stricturoplasty according to aldilaijain. If asymptomatic? No need for


management.
170- A patient presented to the clinic complaining only of perianal discharge (no
abdominal pain or bleeding), on examination three sinuses were noted at the 3 5 7 o'clock
positions. Proctoscopy was clear. What is the most appropriate next step?
A- Colonoscopy
B- MRI
C- Fistulogram
D- US

Depends. Known case of Crohn’s? MRI. Unknown? Colonoscopy.

171- 36 Y.O. male patient k/c of crohn's diseases on infliximab and Azathioprine.
Presented with perianal swelling and tenderness. What is the most appropriate
Management?
A. Increase infliximab dose and decrease frequency.
B. IV antibiotics.
C. Perianal swab.
D. Incision and drainage.

A tender swelling without discharge which is a typical case of abscess. But what if it had
discharge? That would be a complex Crohn’s fistula.

172- A patient with Crohn’s disease treated with azathioprine and infixmab and other
medication presented complaining of pus out from anal. Vital signs stable also no fever.
what is the next step?
A- Pelvic MRI
B- BroadIVAntiBiotic
C- Swabandculture
D- increse inflixmal dose

Normally, we would order an MRI then give abx, and infliximab for refractory cases. But
our patient is passing pus and they are already taking both azathioprine and infliximab
which are immunosuppressants, making it easier for our patient to become septic.
Colorectal
Colorectal screening

Average risk patient


Annual FOBT
Sigmoidoscopy every 5 years
Colonoscopy every 10 years

If positive family hx and a first degree relative was diagnosed before


turning 60 years old -> colonoscopy every 5 years. Otherwise, it’s every 10
years.

In any old patient with GI bleeding or anemia, go for a colonoscopy. If the


case is clear for colorectal then at first you must reach the diagnosis with a
colonoscopy, then CT for staging, then you start treatment. If the patient is
currently obstructed, surgery is indicated to relieve the obstruction before
we do anything else. It’s a general rule in any cancer case.

Right colon cancer -> bleeding/anemia. We do a colonoscopy in an old age


patient if suspected.

Left colon cancer -> obstruction. Relieve the obstruction with a


sigmoidectomy then go for a colonoscopy. No obstruction? Skip to
colonoscopy.

A case of dilated colon without a direct mechanical obstruction? Suspect


Ogilvie syndrome and go for a decompressive rectal tube.

A case of absolute constipation, U shaped colon, empty rectum? Suspect


sigmoid volvulus and proceed with an endoscopy for detorsion then go for
colonoscopy and semi-elective surgery. If failed or unstable? Hartmann’s
procedure.

Mesenteric ischemia
A case of small bowel infarction due to a blockage in the superior
mesenteric artery. Suspect it in patients with ischemic heart disease (or
other cardiac causes like A-Fib) + diffuse abdominal pain for several hours.
Diagnosed with CT-A.
173- 45-year-old women medically free with no personal or family history of cancer. Asking about
when to to start colon cancer screening?
A- no need for screening for her case
B- Start now and every 5 years
C- Start at 50 years with annual colonoscopy
D- Strat at 50 with annual Fecal occult blood

174- 51 years old female medically & surgically free, with no family history of colon cancer, which
colon cancer screening test is appropriate for her?
A. 5 years Colonoscopy
B. Annual Fecal Occult Blood

Colonoscopy every 10 years


Sigmoidoscopy every 5 years
Annual FOBT

175- A 46 years old asymptomatic man. His mother recently died of metastatic colon cancer. She
was diagnosed with colon Cancer 5 years ago at age 69 years. Which of the following is the most
appropriate colorectal cancer screening strategy for this patient?
A- Colonoscopy every 5 years
B- Colonoscopy every 10 years
C- CT Colonography every 10 years
D- Fecal immunochemical testing every 5 years

Again, every 10 years. His mother was diagnosed at 69, but what if they diagnosed her at 59? In
that case, we would go for colonoscopy every 5 years.

176- 43male have family history of colon cancer underwent sigmoidoscopy for polyp removal
Histopathology showed tubular adenoma completely removed ..how to follow?
A-3-6 month
B-3years
C-5 years
D-no need

Repeat colonoscopy in 3 years.

177- What type of polyp with highest risk of cancer?


A. Villous
B. Tubular
C. Tubulovillous

178- After resection of a pedunculated polyp the results was benign adenoma and patient has no
family history of colon cancer what to advice for reduction of colon cancer?
A. Prophylactic sigmoidectomy
B. Prophylactic colectomy
C. Annual colonoscopy
D. Lifestyle modification

179- A 45 years old male smoker has an adenoma removed from his colon. Pathology report
shows a benign lesion. What advice should you give this patient to prevent him from getting
colon cancer?
A. Eat a low-fiber diet.
B. Eat a high-protein diet.
C. Colonoscopy every year.
D. Stop smoking and start exercising
180- An old male patient was admitted as a case of large intestinal obstruction. He underwent
rigid sigmoidoscopy that showed a mass in the sigmoid region. A biopsy was taken and came
back as adenocarcinoma.
What is the best next step?
A - Colonoscopy
B - CT abdomen
C - MRI pelvis
D - Sigmoidectomy

A case of large obstruction, hence why we need to go for surgery first. If the patient is not
currently obstructed and simply has a mass? Colonoscopy.

181- Elderly patient presented to OPD with change in bowel habit (constipation) and streaks of
fresh blood on stool. Rectal exam revealed a mass. Colonoscopy and biopsy confirmed
adenocarcinoma at 6 cm from anal verge. What is the next step in management?
A. Surgery
B. CT chest and abdomen
C. Neo adjuvant radiotherapy
D. Chemotherapy
We did colonoscopy already. Now, we have to stage for metastasis.

182- 65 years man presents to your clinic and looks weak , dehydrated, pale , thin and
emacitaed. he complains of anal itching , discomfort from the pas few months. On
examination, you find an anal mass that is 2 cm away from the anal verge , cauliflower like and
friable. What is your most likely diagnosis?
A- Anal Cancer
B- Rectal Cancer
C- condyloma accuminatae
The keyword is cauliflower like mass.

183- Old male patient lethargic and pale with weight loss for 2 months, by Examination there
was 2nd degree hemorrhoid + low hemoglobin, what’s the diagnosis?
A. Rectal cancer
B. Cecal cancer
C. Hemorrhoids
D. Sigmoid cancer An example of right sided cancer with anemia.

184- Old patient who has constipation on and off with streaking of blood in the stool with no
fulness in the rectum ( no mention of pain )?
A. Sigmoid cancer
B. rectal cancer
C. chronic hemorrhoid. Anemia, and red streaks of blood in the stool due to the location of
D. cecal cancer the cancer.

185- A 45 - year old patient complains of perianal swelling, fresh bleeding per rectum and
weight loss over the last 3 months on examination, there is a mass 1 cm from the anal verge.
She has no obstructive symptoms (see report), Biopsy: Adenocarcinoma. MRI abdomen:
Localized lesion with craniocaudal extension of 3 cm with associated lymphadenopathy.
110/70 mmHg 96 / min 18 / min 36.6 ° C CT scan chest: No evidence of metastasis. Which of the
following is the most appropriate treatment?
A- Diversion colostomy
B- Low anterior resection
C- Concurrent chemoradiation
D- Abdominoperineal resection
In rectal adenocarcinoma, neo-adjuvent chemoradiation is a must regardless of the surgical
procedure.
186- Elderly female, asymptomatic maybe or just fatigued. Labs showed microcytic anemia
what to do NEXT?
A/ Occult fecal blood
B/ Endoscopy & colonoscopy

Old age + anemia = suspect right sided cancer.

187- A 53 year old Male, known to have schizophrenia on medication, presented to ER with
recurrent abdominal distention and constipation. Was normal on examination and vitally
stable.
Abdominal Xray: dilated colon lumen 10cm
CT: No obstruction
Best management?
a-Decompression colonoscopy with rectal tube.
b-emergency colectomy.
c-Lt side colostomy.
d-lower barium enema.

Dilated colon without a mechanical cause. A case of ogilvie syndrome.

188- Elderly with vomiting, constipation, abdominal distention. Upon imaging they described a
shape going towards the right upper quadrant, what’s the dx?
-Rectosigmoid cancer
-Sigmoid volvulus
-obstruction
-closed loop

The shape they are describing is U, in addition to a coffee bean sign on xray.

189- A 65 year old patient known case of Atrial Fibrillation presented to the ER complaining of
severe diffuse abdominal pain for 4 hours. Elevated WBC. Vitally stable. What is the initial test?
A. US
B. CT
C. Diagnostic laparoscopy
D. Unrelated

B, CT-A is more correct.


Most likely a case of mesenteric ischemia although ischemic colitis is another DDx and we use
CT for it. This recall came in my exam.

Another recall:
190- 70 yrs old man k/c of IHD, central abdominal pain vitally stable, amylase 600, WBC normal,
abdominal x ray: dilated small bowl with thickened wall: what’s the
dx?
A. Pancreatitis
B. Intestinal obstruction
C. Perforated ulcer
D. mesenteric ischemia
Hepatobiliary
US is always the first step, even if the patient is obese.
Ascending cholangitis
Charcot triad needs to be present in order to complete the diagnosis: 1)
Fever 2) RUQ pain 3) Jaundice. Treatment is drainage by an ERCP.

Biliary Pancreatitis
It can cause epigastric or RUQ pain. Dilated CBD is key. It doesn’t complete
the Charcot triad, and it doesn’t have features suggesting cholecystitis.
Initially, in pancreatitis cases we order serum amylase/lipase (the latter is
preferred) and an US. In a severe or late presentation, go with CT abdomen
and look for necrotizing pancreatitis.

Cholecystitis
History of biliary colic after fatty meal, fever, and RUQ pain. No jaundice.
No CBD dilatation. Positive Murphy’s sign.
The golden period for cholecystectomy is in the first 72h, but if the patient
is past it manage conservatively then schedule for elective lap chole in 6
weeks.

A symptomatic patient had an US showing sludge? It’s essentially a small


stone so go for lap chole.

Let’s suppose we had an incidental finding of gall stones in an


asymptomatic and stable patient, next? Reassure without further follow up
as this is very common.

ERCP for a dilated CBD is always done before lap chole, but both are done
in the same admission.

Hx of MI patient with a stent? Elective lap chole in 6 weeks.

Lap chole complications


Stable -> US -> drainage -> ERCP definitive
Unstable (or peritonitis) -> laparotomy

Biliary pain in a pregnant patient? Start conservatively if possible, and go


for lap chole if there’s recurrent attacks, uncontrolled symptoms, or no
weight gain. The safest time for lap chole is in the second trimester.

Klatskin tumor -> shrunken gall bladder, dilated intrahepatic ducts.


Pancreatic cancer -> enlarged gall bladder, dilated intrahepatic ducts.
Ampullary cancer -> dilated extrahepatic and intrahepatic ducts.

Gallstone illeus presents with abdominal pain and hx of gallstones years


ago. Classic triad of:
1) Pneumobilia
2) Small bowel obstruction
3) Ectopic gallstone
Hemochromatosis
Ferritin >300
Transferrin saturation (TSAT) >45%
Elevated serum iron
Decreased TIBC

Alcoholic hepatitis
Very high AST and ALT
AST/ALT ratio 2:1

Case of diabetic pt with no lab values? Go with NAFLD as it’s


very common.

Primary biliary cirrhosis -> antimitochondrial antibodies (AMA)

Autoimmune hepatitis -> antinuclear antibodies (ANA), and


anti-smooth muscle antibodies (ASMA). High AST ALT.

Primary scelrosing cholangitis -> associated with UC


(diagnosed by colonoscopy if suspected). Go for MRCP to
confirm.

HCC
Positive antibody but negative RNA?

- If no symptoms, no recent exposure, no abnormal labs, then


no need for follow up.
- If there is a possible recent exposure, repeat RNA in 6
months.
- If the patient is currently symptomatic repeat RNA now.

Screen for HCC with US, then confirm the diagnosis with
triphasic CT. Regular follow up every 6 months with an US if
cirrhosis is present.

Treatment
5cm+ or multiple nodules, go with TACE. Otherwise, go with
surgical excision.

C/I for a liver transplant:


1) Active alcoholic/substance abuser
2) Extrahepatic malignancy within 5 years
3) Advanced cardiopulmonary disease
4) Active uncontrolled infection

Liver mass
The most important step is to avoid contact sports in
hemangioma. Female patient taking OCP? It’s an adenoma so
stop OCP. If neither are present in the choices, then go with
decrease carbohydrates and fatty meals.
191- Patient obese, history of jaundice for 1 week with anorexia and abdominal pain,
examination showed
right upper quadrant tenderness, no history of medication or disease, what is initial step?
A. MRCP
B. Abdominal US
C. CT
D. Biopsy

192- 45-year-old patient complains of recurrent episodes of upper abdominal pain for the
past 2 week with fever. On examination, the patient is jaundiced, the abdomen abdomen is
soft with marked tenderness all over Blood pressure 90/60mmHg Heart rate 130 /min
Respiratory rate 18 /min Temperature 38 °C indirect bilirubin 5 Direct bilirubin 20 Total
bilirubin 25 Alkaline phosphatase 450 Amylase 1400
Ultrasound: CBD of 1.4 cm with dilatation of intrahepatic ducts.
Which of the following is the most likely diagnosis?
A. hepatitis
B. cholangitis
C. pancreatitis
D. acute cholecystitis

Charcot triad. Cholangitis is treated with an ERCP.


193- 46 y.o male present with epigastric pain
heavy dinking alcohol,
Exam :mild epigastric tenderness
previous multiple hospitalization
Labs : amylase normal
Bp. 110/69
Hr 110
Dx?
A- Acute pancreatitis
B- chronic pancreatitis
C- esophgial varisis
D- pancratic psodocyst

Heavy alcoholic and multiple hospitalizations.

194- RUQ pain, for 12 hours, no fever, no jaundice. U.S findings “ non thickened G.B wall
with multiple gall stones, CBD is obsecured “, what’s your diagnosis:
A. Acute Pancreatitis
B. Obstructive jaundice No Charcot triad, and the findings are not relevant for
C. Acute Cholecystitis cholecystitis. Biliary pancreatitis would be more accurate.
D. Ascending cholangitis

195- A 37-year-old woman presents to the ER with a history of right upper quadrant pain
side hours with nausea and vomiting. On examination there was a marked abdominal
tenderness. Abdominal ultrasound demonstrates peri-cholecystic fluids, thick gallbladder
wall with positive sonographic Murphy's sign. Which of the following is the recommended
treatment?
A. Conservative treatment and interval cholecystectomy
B. Insertion of percutaneous cholecystestomy tube
C. Emergency laparoscopic cholecystectomy
D. Emergency open cholecystectomy

Classic case of cholecystitis. The patient presented in the first 72h (side = six?) so go for
lap chole. But if past it? Treat conservatively then schedule for elective lap chole in 6
weeks.
196- 30 y/o complaining of abdominal pain radiating to the back associated with vomiting; he
did gastric sleeve 3 months back. US: gallbladder sludge with no stone; normal cystic duct
EBC normal Amylase 700 What is the most appropriate investigation?
A. Endoscopic US
B. Endoscopic sphincterotomy
C. Laparoscopic cholecystectomy
D. Open cholecystectomy

Sludge is a small stone. Treat it just like you would with a typical cholecystitis.

197- Pt has RUQ pain radiating to the scapula, nausea and vomiting,
U/S shows small gall bladder stones & 5 mm polyp, what is the management?
Lap chole.
Observation
Radical cholecystectomy
Repeat US after 3 months
If any of the following are present refer for surgery:
1) POLYP SIZE >1 CM
2) SYMPTOMATIC
3) AGE > 50
Otherwise? No need for follow up.
198- A patient with epigastric pain admitted to hospital, U/S shows gallstones with dilated
CBD. Now he is stable and labs are normal except high amylase 250. What is the appropriate
next step?
A. ERCP
B. CT abdomen
C. cholecystectomy now
D. cholecystectomy after 6 months
ERCP then lap chole in the same admission.
199- Patient develop surgical emphysema neck chest abdomen after ERCP, which organ was
injured or
perforated?
A. Esophagus
B. Stomach
C. Pancreas
D. Duodenum
200- Old patient came for elective cholecystectomy you find out he was admitted 2 weeks ago
in icu for management of MI, what will you do?
A- do it in this admission
B- delay it 6 weeks
C- delay it 6 months
D - no need to do it anymore
201- Post laparoscopic cholecystectomy, a patient was discharged home. He presented 4 days
later with RUQ pain and fever. Examination revealed temperature of 38°C and tender fullness
at RUQ. Labs showed high WBC level and normal Bilirubin and liver enzymes.
Antibiotic therapy was started. What is the next step?
A. Xray abdomen
B. US abdomen
C. CT scan abdomen
D. Tc-HIDA scan
202- A woman post lap cholecystectomy complaining of SOB and ascites confirmed by US.
Management?
A. ERCP
B. Abx
C. Exploration
D. Percutaneous drainage

We did the US in the prior question, so time for drainage.


203- A 41 years old man underwent a laparoscopic cholecystectomy, 5 days back started to
experience severe abdominal pain and distension. Examination revealed a tender and
distended abdomen. Bp: 100\55 HR: 103 Temp:37.9 ,,revealed the presence of ascites. which of
the following is the most appropriate management :
A) ERCP
B) percutaneous drainage
C) Exploratory laparotomy
D) diagnostic laparoscopy

Vitally unstable or has peritonitis? Laparotomy. Definitive? ERCP after US and percutaneous
drainage.

204- A 30-year-old woman who is 10-week pregnant and know case of gallstone presents to
the Surgical Clinic complaining of recurrent attacks of biliary colic for the last 5 weeks.
Which of the following is the most appropriate management?
A. immediate laparoscopic cholecystectomy
B. laparoscopic cholecystectomy after delivery
C. laparoscopic cholecystectomy in 2nd trimester
D. laparoscopic cholecystectomy in 3d trimester
205- A patient is presented with jaundice. U/S shows a shrunken gallbladder with
dilatation of intrahepatic ducts. What is the most likely diagnosis?
A- Acute Cholecystitis
B-Gall Bladder Stone
C-CBD Stone
D- klatskin tumor
206- A 50 year old man presents with progressive jaundice, dark urine, and right upper
quadrant pain and distention. On physical examination he has a palpable gall bladder. Imaging
shows an enlarged gall bladder and dilated Intrahepatic duct. Amylase = 481. diagnosis is
A. klatskin tumorn
B. Pancreatic cancer
C. Cholecystitis
D. Mirrizi’s syndrome

207- elderly, epigastric pain for 3m, wt loss, jaundice and dark urine
There was dilatation of intrahepatic and extrahepatic duct
Labs: cholestatic picture
(They did not mention if the gallbladder is palpable or shrunk :) )
A. Klatskin tumor
B. Gall bladder cancer
C. Ampullary cancer

208- There was a question about a lady known case of gallstones for 15 years presented with
abdominal pain
and fatigue There was air in biliary system, what is the diagnosis?
A. Gallstone ileus
B. Acute Cholecystitis
C. Acute pancreatitis
209- Elderly with abdominal pain amd abdominal distention, He shows signs of obstruction, on
imaging there is air fluid levels and pneumobilia (air in biliary ducts), What's the next step in
investigation?
A- Gastrograffin test
B- Barium swallow
C- Abdominal CT
D- US

Go for CT in gallstone illeus.


210- A case of young male asymptomatic presented abnormal LFT. He is a smoker and drinks
alcohol in the weeknds. Labs showed high AST ALT (2:1 ratio) Tbilli and slightly high iron and
TIBC and very high ferritin (450). What is the cause of his abnormal LFT?
A-hemochromatosis
B-alcoholic hepatitis
C-cholangitis
AST/ALT ratio 2:1? Alcoholic hepatitis. If the labs simply showed high ferritin the answer
would be A. Now, what if we had no lab values at all in the question in a case of a diabetic
patient? Go for NAFLD.

211- Patient known case of ulcerative colitis did MRCP and showed intra and extra hepatic
duct strictures,
what is the diagnosis?
A. Cirrhosis
B. Primary biliary cholangitis
C. Primary sclerosing cholangitis
First, we have to do an US. Afterwards, for further workup it depends:
Autoimmune hepatitis -> ANA, ASMA
Primary biliary cholangitis -> AMA.
Primary sclerosing cholangitis -> MRCP

212- Young female suddenly developed jaundice and fatigue. She high ALP and high bilirubin.
No splenomegaly or hepatomegaly. US: no finding.
MRCP: multiple foci of stricture and dilatation.
What is best initial next step?
A. Liver biopsy
B. Antinuclear antibody
C. Repeat US
D. Colonoscopy
This is a very similar case. We should look for ulcerative colitis due to the association between
the two.

213- patient admitted in CCU after MI, complicated by pneumonia, during admission he had
RUQ pain, US showed pericholecystis fluid with thick GB wall, Mx?

US guided cholecystostomy tube


Urgent open cholecystectomy

This ICU patient is unable to tolerate lap chole, so go for cholecystostomy tube.
214- 73 years old patient presented with new onset jaundice , weight loss and other
symptoms , CT shows lesions in 70 % of the liver , most appropriate investigation :
1-Colonscopy
2- Upper GI endoscopy
3- Percutanous liver biopsy
4- Diagnostic laproscopy
A case of metastatic liver cancer. Colorectal cancer is the most common cancer metastasizing
to the liver.

215- Alcoholic, right hypochondriac pain and i think weight loss ,alpha fetoprotein is high No
other labs ,Image: CT showed multiple lesion in liver and cirrhosis
A-Hepatocellular carcinoma
B-pancreatic cancer
216- A case of chronic Hepatitis C presents with RUQ mass. Investigations show 6 x 6 cm
hepatocellular carcinoma. What is the best management?
A - Chemotherapy
B - Radiotherapy
C - Transcatheter arterial chemoembolization (TACE)
D - Surgical resection

217- known case of liver cirrhosis secondary to Hepatitis C has completed treatment.
Hepatitis C RNA is negative. How will you follow up this patient?
a. Regular screening with AFP
b. Regular screening with ultrasound
c. Liver biopsy
d. No follow up is required

Due to the presence of cirrhosis.

218- case that was postive for Hep C, came now for followup, labs show HCV Antibody + and
HCV RNA negative
What to do ?
A- Liver biopsy
B- Liver ultrasound
C- Repeat HCV RNA after 6m
D- No need for further intervention
If symptomatic, repeat RNA now. If the patient had recent exposure,
repeat RNA in 6 months.
219- A patient with cirrhosis and his dr. want to screen him for Hepatocellular carcinoma, what
is the best diagnostic test?
Abdominal US
Triphasic CT

Screening? Ultrasound. Confirmatory? Triphasic CT. Could be a bad recall, but since it
mentioned screening specifically the best way is with an US. Another recall: What is a risk
factor for HCC? Hepatitis B.

220- Smoker and obese female patient on combined OCP, at imagining there is 4x4 cm hepatic
hemangioma. What is the most important thing to advise the pt.?
A. Decrease high carbohydrate and fatty meals
B. Stop smoking
C. Eat diet rich in fiber
D. Stop OCP
221- Smoker and obese patient, at imagining there is 4x4 cm hepatic hemangioma. What is the
most important thing to advise the pt ?
A. Decrease high carbohydrate and fatty meals
B. Stop smoking
C. Eat diet rich in fiber
D. Avoid excessive sport
222- 50 years old with Ischemic Heart Disease (IHD), Diabetes mellitus (DM) Admitted to ICU
with severe pneumonia and was treated with Antibiotics. After 3 days of admission, he
developed hypotension and treated with hydration and inotrope, on admission lab was normal
After 3 days, LFT was abnormal Total Bili is 20 (increased), very high AST and ALT (1000),
mild increase in
LDH, US done and showed unremarkable findings, what is the diagnosis?
A. Ischemic hepatitis
B. Intravascular hemolysis
C. ICU related jaundice
D. Acalculous cholecystitis
Pancreatic pseudocyst
We suspect pseudocyst in cases with a history of pancreatitis weeks ago
followed by epigastric pain that makes the patient return. Regardless, the
investigation of choice is always CT scan even if asked for an initial
investigation. US is wrong.

We typically observe these cases, but if one of the following is present go for
endoscopic drainage.

6+ weeks history or 6cm+ collection.

Pseudocyst -> homogenous fluid. Endoscopic drainage. Most common


presentation.
Walled off necrosis -> heterogenous fluid. Percutanous drainage as it’s infected.
Pancreatic abscess -> presents with fever. Percutanous drainage.

If the question doesn’t specify what kind of collection and fluid? Go with
endoscopic (internal) drainage because that’s the most common scenario.

The only exception to the 6 weeks / 6cm size rule is in infected cases. Go for a
percutaneous (external) drainage regardless of how long it’s been or how big
the collection is because the patient is currently infected and needs
intervention.

Liver abscess
Hydatid liver disease
Organism: Echinococcus
Initial: Albendazole
Definitive: Surgical deroofing (especially if daughter cysts are
present).
Amebic liver disease
Organism: E. Histolytica. History of traveling to India/Mexico or
another endemic area is an important keyword. Solitary
hypodense cystic lesion. Amebiasis also causes dysentery
(bloody diarrhea) if it infects the intestines, so that’s another
keyword. Patient may be febrile for a long duration. Treated with
Metronidazole.

Pyogenic liver disease


Most common liver abscess in developed countries. Features of
thick walled hypodense fluid cavity on CT. Occurs secondary to
infection, most commonly cholangitis. May have elevated liver
enzymes. Initiate management with a broad spectrum antibiotic in
all patients. Most patient may require percutaneous drainage as
well.
223- Picture of Two huge echinococcosis hydatid cyst but size wasn’t mentioned, asking
what most appropriate initial step?
A. Albendazole
B. Surgical deroofing
C. Percutaneous drainage
D. Liver resection

224- scenario about hydatid cyst 10*13 with daughter cysts, what is the management?
A. Surgical deroofing
B. Percutaneous aspiration
C. Right hepatectomy

225- 36 old male at ER C/O Right abdominal Pain, O/E: fever, anorexia, weight loss,
tenderness in RQ and Lower intercostal margines also patient is toxic
Temp. 37.9 (I think but it was elevated) wbc high, bilirubin high US: cystic lesion without
septates CT : homogenous (not sure) and "THICK WALL with
Peripheral enhancement
- What's most appropriate
A. Ceftriaxone
B. Metronidazole Case of pyogenic liver abscess, which is the most common
C. Surgical drainage cause of liver abscess by far in developed countries. The most
D. Percutaneous drainage appropriate management is drainage. As for the initial step, go
with Ceftriaxone since we have to start it in all patients in
addition to drainage.

226- Male patient came from India RUQ pain .. on and off fever for 3 weeks . raised LFT , high
WBC (Neurtrophol 70% Lymphocytes 20%) . image showed homogenous hypoechoic mass in
the liver.
a) hydatid cyst
b) TB abscess
c) amebic abscess As mentioned previously pyogenic abscess is the most common
d) pyogens abscess one, but since the patient came from India and has been febrile for
3 weeks we will go with amebic abscess.

227- 45 yo man with returns from a trip to Mexico and develops fever, chills, and RUQ pain.
WBC count is 20. US shows an intrahepatic fluid collection. CT scan shows 12 cm single
abscess with a peripheral rim of edema. This condition is best treated with?
A. Percutaneous drainage and Abx
B. Metronidazole
C. Albendazole Mexico is another endemic area for amebic abscess. What if we
D. Surgical drainage removed it from the question? The answer would change to A.

228- 7 YO male presented to the hospital complaining of fever, bloody stool and tenesmus
for 3 days. Abdominal examination reveled abdominal distention.
What is the most likely diagnosis?
A. Ascaris
B. Amebiasis
C. Giardiasis
D. Rotavirus
229- Case of Pancreatitis 5 weeks ago. Now she has epigastric tenderness and cannot tolerate
food with vomiting each time. By ultrasound, you found large about 12x10 cm mass with thick
wall and fluid inside which is heterogenous and non-liquefied. Labs: 346 amylase, WBC 15000.
What is the diagnosis?
A. Pancreatic Pseudocyst
B. Pancreatic Abscess
C. Walled off pancreatic necrosis

Heterogenous fluid, making it a case of walled off necrosis. If homogenous fluid? Pseudocyst.
If febrile? Abscess. What if none of it was mentioned to begin with? Pseudocyst since it’s the
most common presentation.

230- Pancreatitis 5 weeks ago. Now she has epigastric tenderness and cannot tolerate food
with vomiting each time. By ultrasound you found large about 12X10 mass with thick wall and
fluid inside. Labs: 346 amylase, Wbc 15k. What is the diagnosis?
A. Pseudocyst
B. Abscess
C. Walled off necrosis
(Didn’t specify homogenous OR heterogenous)

No fever, and no mention of a heterogenous collection.

231- 20 Patient diagnosed with acute pancreatitis 3 weeks ago, now present to ER Complain of
mild abdominal pain and tenderness, US showed cyst measure 4 cm, how you will manage?
A. Observation
B. Internal drainage
C. External drainage
D. Surgical remove

3 weeks and 4cm, so keep observing for now. Afterwards, go for internal drainage.

232- Case of pancreatic cyst for 5 Ws with collection was 18 cm x 24 cm how to manage?
A- Percutaneous drainage
B- Endoscopic drainage
C- Surgical drainage

No additional details that would change the answer (fever/heterogenous), so go for


endoscopic drainage.

233- one Q was typical pancreatic pseudocyst, diagnosis mentioned in the Q, also the patient
was observed for 6 weeks and the cyst getting bigger, what is appropriate management?
-laproscopy drainage
-percutaneous drainage
-excision of the cyst
-I dont remember maybe it was observe

Bad recall. There are two possible explanations:


1) the 4th option is endoscopic drainage, which would make it the correct choice since
we always manage pseudocysts through an internal drainage.
2) if the 4th option is unrelated, the other possibility is that the recaller forgot to
mention some important details in the question. Fever? Heterogenous fluid? The
answer in that case would be percutaneous drainage.
Post-op infection
SSI approach
Open wound -> CT to assess for possible deep infection ->
percutaneous drainage

Intra abdominal collection


Abx if small size, percutaneous drainage if 4*4 or bigger,
laparoscopy if multiple collections, and laparotomy if the
patient is unstable.

“Open drainage” isn’t the same as open the wound. It refers


to percutaneous drainage which is done after CT, so be
careful. The proper way to drain percutaneously/externally
is through CT-guided drainage.
Open the wound = remove stitches/clips = I&D.

A case of abscess? Simply go for an incision & drainage.


234- Patient with perforated appendicitis after surgery had pus from wound, pain
localized to the surgical site. No guarding no fever what best initial treatment is:
A. Antibiotics
B. Open drainage.
C. Imaging guided drainage
D. Wound exploration

235- 25 year old male Pt 8th day


post surgery with wound site redness & tenderness with purulent discharge.. most
appropriate?
A. IV antibiotics
B. CT abdominal
C. open drainage
D.exploratory laparoscopy

236- A 42-year-old woman underwent an uneventful laparoscopic cholecystectomy, 2


weeks later, she present to the Emergency Department with vague abdominal pain. CT
scan: Large collection in the subhepatic area. Which of the following is most
appropriate next step?
A. Operative drainage
B. CT-guided drainage
C. Laparoscopic drainage
D. ERCP with biliary stent placement

237- Post appendectomy female came with LR abdomen mild tenderness Ex Normal
By CT there is 2*2 collection in Retrocecal :
A. Exploring laparotomy
B. percutaneous drainage symptomatic
C. laparoscopic
D. conservative with Anitbiotic

238- A 36 y.o male known case of crohns for 10 years, presented to ER C/O
abdominal pain, fever, vomiting and diarrhea, O/E there is abdominal tenderness.
CT showed: 12×15 collection and ileo-jejunal fistula. How to manage?
A-Laproscopic drainage
B-Percutaneous drainage
C-Open drainage
D-Open drainage with fistula
resection.

239- Patient post appendectomy, came for regular follow up post-surgery, no active
complaints, on exam he
has seroma which drains freely from the opening of the wound, no erythema no pain
no fever, what is the
appropriate management?
A. Observation
B. Open wound exploration
C. Regular wound dressing
D. US guided drainage

A case of a serous fluid collection called seroma.


ABC
A
1) O2 sat 88%
2) GCS 8
3) Unconscious
All are indications to intubate.
In addition, we use an oxygen mask as pre-oxygenation for a
conscious alert patient, but the best way is orotracheal intubation if
the patient is unconscious or moaning. Let’s suppose the same
patient had a face fracture (mandible), in that case go for a
cricothyrotomy.

B
Typically in cases such as tension pneumothorax which is treated by
needle decompression. Never choose needle decompression before
intubation as it’s wrong in the SMLE despite what UTD says. Follow
the sequence.

C
IV fluid and circulation control, then proceed with imaging if no
disabilities exist.

Always follow ATLS in every case before choosing anything else.

Severe vomiting -> hypochloremic hypokalemic metabolic alkalosis


(loss of Cl and K). Treated with normal saline.
Severe diarrhea -> metabolic acidosis (loss of HCO3 and K). Treated
with ringer lactate.

Both hypokalemia and hyperkalemia affect the heart rhythm.


Hypokalemia (Flat T wave) is treated with oral potassium.
Hyperkalemia (Peaked T wave) is treated with calcium gluconate, and
the second line is SABA/Insulin. Dialysis for refractory cases.

Severe hyponatremia (NA below 120) -> neurological symptoms.


Treated with hypertonic saline.

Feeding
If the patient doesn’t have an adequate oral intake, we need to
provide nutrition in an inpatient setting. First, check the GI function, if
it’s not functioning properly start with parenteral nutrition (IV access).
Otherwise, enteral feeding is indicated. An acute case below 1
month?
Start with NGT. Risk of aspiration? Go with NJT. Both are initial
measures if the question asked for such.
A chronic case more than 1 month?
Initiate resuscitation with gastrostomy. Risk of aspiration? Go with
jejunostomy. Both are considered the most appropriate management/
best in a chronic case.

Wound closure
1) Open scalp laceration within 6 hours? Primary closure/suturing as
it is within the 24 hour window.
2) Injury reached the tendons and nerves? Primary repair of
structures is indicated.
3) Exposed necrotic skin? Secondary closure.
4) What if it’s not infected? VAC.
‫اﯾش ﻣﺻدرك ﻓﻲ اﻟﺟراﺣﺔ؟‬
‫ ﻛﺎاﻓﻲ‬UTD ‫ﯾﺎ رﺟﺎل‬

Schwartz,
Washington
manual

B
Typically in cases such as tension pneumothorax
which is treated by needle decompression. Never
choose needle decompression before intubation
as it’s wrong in the SMLE despite what UTD says.
Follow the sequence.
240- Patient with stab wound in anterior neck, he is Alert but in labs oxygen sat 82%. What to
do?
A. Oxygen mask
B. Cricothyroidotomy
C. Endotracheal intubation
D. Tracheostomy

As explained earlier. Also, in the full recall they asked for next.

241- Patient with a stab wound in the right thigh and massively bleeding brought to the
emergency department unconscious and O/E there is active bleeding. What is the most
important next step to do ?
A. Ringer lactate iv fluid
B. Blood transfusion
C. Tourniquet on the thigh
D. Orotracheal intubation

This patient has problems in both A and C. Prioritize the airway and go for an endotracheal
intubation since the patient is unconscious.

242- Patient involved in Road Traffic Accident (RTA) with multiple mandibular fracture with
sever bleeding, unconscious, no vitals mentioned How would your mange his Airway?
A. Laryngeal mask
B. Orotracheal
C. Nasotracheal
D. Cricothyrotomy

Because of the face fracture.

243- A child 10 y was beaten many times on the face during a fight. A trial of endotracheal
intubation has failed.
How will you secure the airway?
A - Tracheostomy tube
B - Cricothyroidotomy
C - Orotracheal tube
D - Nasopharyngeal airway

Similar question but in a pediatric age group. A according to Thawaba.

244- 35 year-old car driver crashed into a concrete block without a safety belt on. Thirty
minutes after and on the way by ambulance to the hospital he begins to become breathless.
On administration of 100% oxygen there is not much improvement in this condition. On
arrival at the Emergency Department he has lost consciousness and appears cyanosed with
markedly distended jugular veins. Blood pressure 80/40 mmHg Heart rate 120/min
Respiratory rate 34/min Temperature 36.6°C Oxygen saturation 60% on room air What
immediate action should be taken?
A Intubation and 100% oxygen
B. Rapid infusion of crystalloid
C. Needle decompression
D IV 0.2 mg adrenaline

Always go with ABC in the SMLE. Intubate then go for needle decompression.
245- 100- Men had multiple gun shots bleeding from
everywhere He is oriented opening his eye spontaneously
obey command Vitals
80/60
Pules 133
RR 25
O2 88%
How to manage him?
1- inserting 2 iv line
2- laparotomy
3- oxygen mask
4- CT

Why rush to laparotomy? The patient is desaturated.

246- 15 years old boy, was in burning house, He has carboneous sputum, hoarseness of the
voice, lung crackle or creptation , whatis the most likely cause of his presentation?
A- inhalation injury
B- carbon monoxide toxicit
247- Patient came to ER after history of building burn, patient confused with burn of facial and
nasal hair, what to do:
A- elective intubation
B- ICU observation
C- advice patient to take analgesia

248- Pt had RTA in er, only has a bruises in lower abdomen and abdominal pain, what is the
most appropriate nest step ?
Bp : 110/80
Temperature : 37.2
HR : 88
RR : 19
A. CT
B. Fast
C. Initial assessment and resuscitation
D. Exploratory laboratory

249- middle age found to have euvolumic hyponatremia , and small lung cancer ..all labs
normal except Na was 115
what’s the type of fluid replacement will give?

A - normal saline
B - half NS
C - hypertonic saline

250- Hyperkalemia 6.5 in CKD patient,


next step ?
A. Calcium gluconate
B. insulin and salbutamol
C. Dialysis

Indication of hyperkalemia treatment:


1) 6.5+ K
2) Peaked T wave on ECG
A initially, then B, then C for refractory cases.
251- Male patient came to ER with right femur shaft fracture, Hemodynamically stable, no other
injuries. what
is your priority?
A. Pain management
B. Blood control
C. Maintaining function
D. Decrease soft tissue trauma

We need to maintain the circulation, the rest are not as important.

252- Adult fell from height complains of severe heel pain. He is conscious, oriented and has
stable vitals. What
is the next step?
A. Pain control
B. Lower limb X-ray
C. Pulse palpation

Primary survey. A simple and fast physical exam should always be the first step.

253- Case of anterior thigh deep laceration with heavy bleeding and unstable patient asking
about what to do next?
A- Direct pressure on the wound
B- pressure above the wound on femoral artery
C- apply tourniquet

254- A case of basal ganglia stroke, with difficulty in swallowing and absent gag reflex. History
of losing weight. How to provide nutrition?
A. Gastrostomy
B. Jejunostomy
C. NGT
255- Pt with basal ganglia bleed and decreaed gag reflex, most appropriate way to initiate
nutrition:
- gastrostomy
- Jejonostomy
- Paranteral nutrition
- NGT

Different scenario. The first one is most likely asking about the most appropriate method to
provide feeding for the patient (not initial), and the second question is asking us the most
appropriate way to initiate feeding. NJT would be better than NGT since the patient is at risk
for aspiration. It depends on the full recall in the exam, so be careful.
256- Male patient came with scalp open wound, after 6h assault, what wound management?
A. Secondary closure
B. Debridement with Primary closure
C. Debridement with granulation
D. Leave it for granulation

Primary suturing/closure as it’s within the 24 hour window.


257- A patient presented with a knife injury to the hand. O/E: the laceration reached
the tendon and nerve. How will you repair this injury?
A. Primary repair to injured structures
B. Debridement with primary closure
C. Debridement with secondary intention
D. Debridement with Vacuum assisted closure (VAC)

Primary repair to injured structures as it reached the tendons and nerves.


258- An elderly patient in ICU has bed sores with exposed necrotic skin and subcutaneous
tissue. What is your
management?
A. Debridement with a skin graft
B. Debridement with assisted vacuum
C. Debridement with primary closure
D. Debridement & Dressing with secondary closure

Exposed necrotic skin, so go with secondary closure.

259- Few days after laparotomy, patient had pus discharge from the wound, for which it was
opened to allow drainage. Few days later, wound looked healthy and with good granulation
tissue. It was large and deep with intact abdominal fascia, no signs of infection. Most
appropriate management?
A. Open the fascia
B. Antibiotics
C. Place a vacuum-assisted closure (VAC) device
D. Wound debridement

VAC. No signs of infection.


Lung nodule approach

First, check if a previous chest X-ray exists.


If the lung nodule hasn’t changed in 2-3 years, then no need
for further follow up.

More than 2cm -> surgery


Between 0.8mm and 2cm -> PET or Biopsy
More than 0.4mm but less than 0.8mm -> CT scan follow up
0.4mm and below, no need for follow up
260- Lung node meassure 8mm asymptomatic what to do?
A) Follow up
B) CT scan
C) biopsy
D) review previous x-ray

Always the first step.

261- 72 old male with 8 mm nodule in lung discovered 3 years ago incidentally
Without symptoms. He did follow up for the last 2 years and also this year no change
in the size and no symptoms what to do?
A. Pet scan
B. Follow up
C. No need to follow Up

No change in the size for 2 years.

262- 32 non smoker presented with lung nodule 7 mm otherwise normal except
for ca 4 mmol :
A. Repeat CT scan within 3-6 months
B.Pulmonary funtion test
C. Biopsy of the lesion
D. PET scan

0.5mm - 0.7mm = CT
If 0.4mm or below? No need for follow up.

263- 55 yrs Patient is having 1cm lung nodule with ct evaluation for abdominal
pain. patient has no respiratory symptoms. Patient is ex smoker for 10 years
that stopped smoking 20 yrs ago
A. Ct 3-6 months follow up
B. Surgical resection
C. Transbronchial biopsy
D. No follow up

If you see both PET and Biopsy in the choices, go with PET. Patient stopped
smoking 20 years ago? Irrelevant detail which does not change anything as the
nodule is still 1cm.

264- Female smoker 50 pack/year had a lung nodule that was 5mm and after 9
months it became 10.5mm. She has no symptoms, normal physical examination , no
LN enlargement, nothing at all. What to do next?
A. blind percutaneous biopsy
B. Refer to thoracic surgery
C. Bronchoscopy
D. follow up after 6 month

Because the nodule is rapidly growing in such a short time, it would be best to refer
the case.
Obesity
Bariatric surgery is the best way to reduce weight.
Indicated in a BMI of 40+ or 35+ in a co-morbid patient.
You may have to calculate it yourself. The next best way
after surgery is lifestyle, then Orlistat.

To choose the type of procedure, we perform an


endoscopy.

Hx of roux-en-Y bypass -> internal hernia


Hx of sleeve gastrectomy -> adhesions

Patient post bypass surgery? Go with CT even if


unstable.
Patient post sleeve surgery? Go with endoscopy.

Jejunum -> folic acid absorption


Terminal ileum -> vitamin B12 absorption

If either of the two are tampered with in a bypass


surgery? It eventually causes megaloblastic anemia, so
those patients need parenteral replacement.
265- A female patient went to obesity clinic for advice regarding surgical methods of
weight loss. After full explanation by the surgeon, which of the following is the most
efficient/effective way of weight reduction?
A) Intensive Exercise
B) Lifestyle Modifications
C) Orlistat

The very best is surgery, followed by lifestyle modification. The third best is a medication
called Orlistat. Remember, we can only offer bariatric surgery in patients with a BMI of
over 40, or over 35 if co-morbidities exist.

266- Patient Obesity wants weight reduction no other conditions all normal question ask
what is the most important to do to decides the type of surgery?
A. Barium swallow
B. Ct abdomen
C. Ultrasound abdomen
D. Upper gi endoscope

267- Patient came after 3 days after Roux-En-Y surgery complaining of fever chill and left
shoulder pain, best diagnostic investigation?
A. CT with contrast
B. Endoscy
C. Laparoscopy
D. Exploratory laparatomy

A, even if the blood pressure is 90/60 in another recall. If the patient presented after
sleeve, go with endoscopy.

268- 29-year-old lady presents with central abdominal pain for three days, nausea and
vomiting for one day. Her surgical history includes laparoscopic sleeve gastrectomy 6
years ago. Physical examination reveals dehydrated with distended abdomen and
exaggerated bowel sounds.
What is the likely diagnosis?
A - Incisional hernia
B - Internal hernia
C - Intussusception
D - Adhesion

Hx of sleeve. If hx of roux-en Y, choose internal hernia.

269- patient 3 days post sleeve gastrectomy, he is presented with mild RUQ pain,upon
examination there was mild tenderness with no signs of peritonitis vitals: BP:100/80
HR:133 what is the cause?
A- gastric leakage
B- sepsis
C- decreased oral hydration
D- inadequate analgesia

A according to Thawaba.
Acute Limb Ischemia
ALI approach
Heparin - US - CT-A (skip if paralyzed limb/class III) -
Angiography

PAD approach
Heparin (only if acute on chronic, skip otherwise) - ABI - US -
CT-A (diagnostic only) - Angiography for revacularization as
the definitive management and investigation in CLI.
A supervised exercise program to increase the walking
distance, and smoking cessation is a must in PAD. Aspirin to
prevent cardiovascular events.

Absent pulse is the first sign in all classes of ALI.

Class 1 -> Moderate pain


Class 2 -> Severe pain, moderate sensory loss and
paraesthesia
Class 3 -> No pain, paralyzed limb

Treatment
Choose amputation if it’s an acute on chronic case with an ABI
less than 0.3 or if irreversible paralyzed limb (class III),
embolectomy if A-Fib or other cardiac cause. Catheter
thrombolysis is used in arterial thrombosis and is also
preferred over embolectomy as the first line treatment in ALI
(IV tPa is wrong in limb ischemia, you only choose it in stroke/
MI/PE).

Foot ulcers
If the pulse is absent, that indicates an arterial issue as
discussed in the ALI section so go with arterial US next. If the
pulse is intact and there is a dark skin discoloration, go with
venous US. The most common risk factor for a venous ulcer is
venous hypertension, followed by age.

AAA approach
Symptomatic and stable - CT-A
Asymptomatic and stable - US (screening)
Unstable and unknown case - US
Unstable and known case of AAA - immediate laparotomy
270- 67 YO male admitted with MI. after two days of discharge he developed severe pain in
his left leg. What is the most likely cause?
A. Acute arterial thrombosis
B. Acute Arterial Embolus
C. DVT
D. Neuropathy

Cardiac = embolus. Acute arterial embolus presents more acutely than thrombosis.

271- 65 YO male presented to ER with acute left lower limb pain for 3 hours. Associated
with numbness and absent pulse. ECG: Atrial fibrillation. What is the best next step?
A. Heparin
B. CT Angiography
C. Arteriography
D. U/S doppler

Cardiac related, so this is caused by an embolus. The best treatment is embolectomy.

272- Patient with chronic limb ischemia, presented with sudden leg pain, diminished
popliteal and distal pulses in right leg, and diminished distal pulse with intact popliteal in
the left, what's the appropriate next action?
A. Heparin
B. CT angio
C. US
D. conventional angio

273- 55 year old male diagnosed with acute lower limb ischemia. I.V heparin andl.V fluid
fluid started. What is the most appropriate next step in this patient management ?
A. СТ-A
B. DSA
C. US-Duplex
D. Immediate embolectomy

US next after Heparinization.

274- Pt had decreased sensation and painful left leg while walking relieved with rest.
Examination: Left leg palpable femoral and popliteal but distal are not palpable.
Right leg: palpable femoral and poplitteal. dista veins are not palpable.
-What is the appropriate investigation?
A) CT angiography.
B ) Vascular US.
C) Conventional angiography.
D) magnetic resonance angiography

Most appropriate investigation (not next step) -> CT-A


275- 65 YO male presented to ER with severe right leg pain and absent pedal pulse.
Which of the following is the most diagnostic investigation?
A. СТА
B. MRA
Most diagnostic.
C. Conventional angiography
D. Ultrasound doppler

276- 39 year old male had a pelvic fracture from MVC rollover 2 months back that treated
surgically. Before 2 hours he started to complain of sever left LL pain. Femoral pulses are
intact. I.V heparin started and venous US is negative. What is the the best management?
A. Catheter thrombolysis
B. Embolectomy
C. Observation
D. Amputation

Catheter thrombolysis is preferred over surgery unless they give history of a cardiac cause.

277- 61 year old female known case of A. Fibrillation. Presented to the emergency
department with 3-4 hours history of sever leg pain. On examination palpable femoral pulse
and absent popliteal. Associated with diminished sensation and altered motor function.
What is the most appropriate management ?
A. Amputation
B. CT-A
C. Embolectomy
D. Catheter thrombolysis

As explained earlier, cardiac cause? Embolectomy.

278- Old patient long history of leg claudication *for 2 months* , present with leg pain and
ABI <0.3 and , *CTA show artery occlusion more than 3 cm* what to do?
A- amputation
B- thrombolysis
C- embelctomy
A case of acute on chronic with ABI of less than 0.3 indicating critical ischemia. Chronic
history of PAD followed by an acute episode of ALI. If the ABI was higher, choose
thrombolysis.

279- Diabetic patient walks for 300 meters then feels pain in his legs and must rest, What
probably is the causes?
A- Varicose vein
B- Arterial causes
C- infection
D- Diabetic neuropathy
Clear case of peripheral artery disease.
280- Elderly male known case of high blood pressure presented with lateral
malleolus ulcer. What is the first test to do ?
A. CT-A
B. ABI
C. US duplex
D. VT venogram

Start with an ABI in PAD then go for an US.

281- DM with hx of pad, had pain when he walks 300m and relieved by rest what
will you do
A- CTA
B- vascular ultrasound
C- conventional Angio
D- MRA

US next, angiography best.

282- Patient with PAD has 100-meter claudication's, DM, heavy smoker, not getting
better. How to improve his walking distance?
A. Supervised exercise program
B. Strict glycemic control
C. Smoking cessation

A to improve his walking distance specifically, but smoking cessation is superior


overall.

283- Patient knowing for DM And HTN and history of right leg pain increased by
exertion, on examination absent popliteal pulse on right leg
Which one of the following indicates acute limb ischemia:
A-intermittent claudication
B-rest pain
C-scar for iliofemoral bypass in left leg
D-swelling

Rest pain goes with ALI. Intermittent claudication goes with PAD/chronic limb.

284- 35 years old patient come with medial leg ulcer. The most likely diagnosis is:
A- Diabetic
B- Venous hypertension
C- Atherosclerosis
D- Buerger's disease.

285- 56 years old woman presents to the clinic with a non-healing ulcer over her
right lateral malleolus, she is hypertensive. pulse is normal and her local exam
shows dark discoloration of the skin around the ulcer
and viable ulcer bed, best next step is?
A. CT angiogram
B. Venous duplex US
C. Arterial doppler US
D. Conventional angiograph
US initially. In this case, venous US because of the dark discoloration and a viable
ulcer bed.
286- Elderly smoker known case of poorly controlled DM comes with ulcers on tip of three
of his toes, diminished dorsalis pedis bilaterally, however, intact popliteal pulse, what’s the
initial management?
A. Amputation
B. Long term anticoagulation
C. Immediate surgical intervention
D. Diet modification and lifestyle changes

If dietary modification isn’t in the choices, go with debridement.

287- Diabetic patient with pseudo hyperepithelialization in situ, what should we do?
A. Amputate toe
B. Ulcer debridement
C. Follow up A biopsy showing pseudo hyperepithelialization could be
D. Repeat biopsy. squamous cell carcinoma in reality, so we have to make sure.

288- Case of Abdominal Aortic Aneurism (AAA) and the patient is hemodynamic unstable,
present after he ate food with severe abdominal pain despite taking analgesic. He became
confused and unconscious later in the hospital. Examination revealed: Tender and pulsatile
mass in the abdomen, BP low. What is your most appropriate management?
A. US
B. CT
C. Exploratory Laparotomy

As a general rule, whenever you encounter a question in the Qbank that goes
like “case of X” it means they were giving you the symptoms, not the diagnosis.
"Known case of" is a different story.
PE approach
1) D dimer initially, CT-A if unstable pt or definitive
2) Compressed US in a pregnant patient, but if it’s negative choose V/
Q scan.

Features of massive PE:


1) Syncope.
2) Raised JVP.
3) ECG showing: sinus tachycardia.

PE Treatment
LMWH or fondaparinux are preferred over UFH
Choose tPa/thrombolectomy if unstable patient or massive/saddle PE
Rivaroxaban if it’s about chronic management

DVT Treatment
LMWH (pregnancy, cancer case)
40mg is prophylactic
40mg BID is therapeutic

UFH (renal impairment)


5k IU BID is prophylactic
10k IU is therapeutic

In severe DVT cases that mimic ALI by having ischemic changes, the
answer changes to tPa, or IVC filter if tPa is C/I (for example stroke, or
the patient is post-op in the recovery room).
289- Pregnancy typical DVT symptoms then developed PE, how to dx?
A- Ct
B- D dimer
C- V/Q
D- Us for lower limbs

Compressed US initially, but if it fails go with V/Q scan as the definitive choice for a
pregnant woman.

290- Patient did heamorrhoidectomy 2 weeks ago now came with sob
( given ecg but was not clear)
What is the confirmatory test?
A..spiral ct
B..echo
C..ecg
D..dimer

In the exam, the ECG was probably showing sinus tachycardia. Go for D if asked for an
initial test.

291- Elderly kc of htn well controlled c/o confusion and irritability. He was fine except when
he injured his left thigh 5days ago and he was bed bound ever since. VS shows hypotension
and o2 of 88% Also shows tachycardia and he has arrhythmia on ecg
A - PE
B - cerebral infraction
C - brain hemorrhage
D - arrhythmia

History of a bedridden patient eventually leading to massive PE with ECG changes and CNS
symptoms.

292- Case of pulmonary embolism , hypotensive pt, and they mention *saddle emboli in CT*
A- warfarin
B- enoxaprin
C- thrombolectomy

saddle = massive. Go for thrombolysis/thromboectomy, whichever is in the choices.

293- 35 year old male have sudden SOB and chest pain was shifted to ICU, diagnosed with
Massive PE (no vitals were mentioned) what is the next initial step:
Heparin infusion
TpA

294- Patient with DVT then developed PE, ( no evidence of renal failure in the case) Which of
the following
most appropriate management
A- Aspirin
B- Foundaparinux
C- IV heparin (no enoxaparin)
D- Revaorxaban

LMWH would be preferred, so with these choices go for B. According to UTD, Rivaroxaban
is more suitable to be used as an indefinite anticoagulant management but for this case we
need an acute treatment. UFH is less useful in PE.
295- A 73 year old woman is brought to ER after a fall at home. She is diagnosed with left hip
fracture (see lab results) Weight 82 kg Sodium 136 (normal) Potassium 4.2 (normal) Creatinine
68 (normal) What is the best order by the admitting orthopedic surgeon to prevent deep vein
thrombosis?
A. Aspirin 81 mg PO daily
B. Enoxaparin 40 mg SC daily
C. Fondaparinux 10 mg SC daily
D. Heparin sulfate 10,000 units IV BID

To prevent DVT, choose a prophylactic dose as outlined earlier.

296- Patient presented with DVT what regimen to use:


A- Aspirin 61 mg
B- Enoxaparin 40 mg SC
C- Fondaparinux 20 mg
D- Hepatin 10,000 U IV

Since the patient already has DVT, we need a therapeutic dose.

297- A 66 years old female admitted to general surgery ward after major rectal surgery, on the
second day she developed leg swelling on the side of the operation. investigations showed
DVT on the femoral vein. the best management to this patient is:
A. LMWH.
B. Thrombolytic therapy.
C. Warfarin.
D. Inferior vena cava filter.

298- Old pt after rectal surgery he is not doing well after in the recovery he start to have leg
pain he developed DVT from the popliteal to the femoral ..
A. Enoxiparen
B. Heparin
C. Warfarin
D. IVC

The keyword is patient in the recovery room. This is a case of PCD (severe DVT with absent
pulse) which is typically managed by thrombolysis but it’s contraindicated due to the recent
surgery within 24 hours.

299- patient with recent rectal surgery comes to you with absent pulses up to the femoral area.
How will you manage such a case?
a. Unfractionated heparin
b. Enoxaparin
c. IVC filter
d. Thrombolysis

A somewhat similar case of post-op DVT becoming severe enough to cause limb ischemia, so
the first line treatment should be thrombolysis. In another recall thrombolysis isn’t available,
so in that case go for heparin.
Orthopedics
Supracondylar fracture approach
Urgent reduction, then K-wire if pink and warm, surgical
exploration if pale and cold. (If you have to choose between the
last two and the question is vague, go with K-wire).

Lower limb fractures


Adults -> Close reduction with intramedullary nail

Children
Less than 6 months -> Pavlik harness
6 months to 5 years -> Hip spica
More than 5 years -> IM nail

Open fracture approach


IV antibiotics -> close reduction -> debridement -> definitive with
IM nail or external fixation if extensive soft tissue damage.

Compartment syndrome
Pain (1st alarming sign), Paraesthesia (2nd alarming sign), Pallor
Pulseless, Paralysis. Urgent fasciotomy.

Pelvic fracture
Stabilize the pt with ringer lactate and a pelvic binder, then head to
the OR.

Fat embolism mimics Pulmonary embolism, but the main


difference is that FE has CNS symptoms and petechial rash.

Most common shoulder dislocation is anterior (Abduction and


external rotation). However, posterior dislocation occurs in
epileptic patients.
Most common hip dislocation is posterior (adduction and internal
rotation).
300- Pt after fell down from hight present with open fracture and dirty , what the most
appropriate next step in the management ?
A- oral antibiotic
B- open fixation
C- surgical debridement
D- external fixation

IV antibiotics, then debridement.

301- 31 years old male was involved in MVA. He had large wound and skin loss. He was
diagnosed to have open left tibia shaft fracture.
What is the definitive management?
A. Close reduction and fixation with Intra-medullary nail
B. Open reduction and fixation with external fixation
C. IV antibiotics, wound irrigation, splinting
D. Wound exploration

Typically, open reduction (not close) and fixation with IMN which is also referred to as
ORIF is the definitive treatment, but in this question we need an external fixation because
of extensive soft tissue damage. C would be initial.

302- 5 years old child present with supracondylar fracture after falling down. He's arm is
deformed, he has no pulse but the limb is warm and pink. What's the next appropriate step
for management?
A. Surgical exploration
B. Urgent reduction and K wire fixation
C. CT angiography
D. Splinting and observation

303- 6 years old child present with supracondylar fracture. He's arm is deformed and he
has no pulse. Reduction and fixation with k-wire done but still there is no pulse and the
limb is pale and cold. What to do next ?
A. Observation
B. Surgical exploration
C. Doppler US
D. CT angiography

Initially, we go with close reduction then the next step depends on the temperature and
color of the skin. But what if the question was vague? Choose K-wire.

304- 3 years old child fell down from a swing and sustained right femur closed fracture
with 30 degree angulation.
What is your management?
A. Traction and observation
B. Application of Pavlic Harness
C. Reduction and fixation with intramedullary nail
D. Reduction and spica cast application

305- 7 years old child fell down the stairs and sustained left femur closed fracture with 30
degree angulation.
What is your management?
A. Traction and observation
B. Reduction and fixation with External fixation
C. Reduction and fixation with flexible intramedullary nail
D. Reduction and spica cast application

306- 30 years old male victim of MVA was brought to the hospital with red crescent. He
was diagnosed with right femur shaft close fracture.
What is the definitive management?
A. Close reduction and fixation with External fixator
B. Close reduction and fixation with Intra-medullary nail
C. Open reduction and fixation with Plate & screws
D. Casting
307- 12 years old obese male presented to ER complaining of left hip pain after falling down.
He can't weight bear. O/E the hip is slightly flexed and externally rotated.
What is your working diagnosis?
A. Hip dislocation
B. Septic hip
C. Neck of femur fracture
D. Slipped Capital Femoral Epiphysis

308- 35 years old male victim of high speed MVA, was brought to ER. He's conscious alert
oriented. BP 100/70. IV fluids was started. He's right lower limb in external rotation. What's
your next step in management.
A. CT pelvis
B. Pelvic binder application
C. Right lower limb skin traction
D. Operative fixation

The patient was given ringer lactate already, so the next step is pelvic binder, and the
definitive is operative fixation.

309- Patient came after limb trauma, with severe pain and paresthesia between his toes and it
was pale. X ray shows fractures, intercompartmental pressure was 35mmhg what to do?
A. Internal fixation
B. Closed reduction
C. External fixation
D. External fixation with four fasciotomies

Another recall:
310- 40 years old male victim of high speed MVA. He sustained Tibial shaft fracture. He is in
severe pain, has absent dorsalis pedis and posterior tibial pulse, numbness in the lower limb
What is the appropriate management?
A. Splint and elevation
B. Urgent fasciotomy and External fixation We only go with C if we are not sure.
C. Compartment pressure measurement
D. CT angiography

311- 25 years old presented to the ER after MVA with fracture of the lower limb. He underwent
reduction and fixation with Intramedullary Nail.
After operation he became confused and developed dyspnoea, petechiae and tachycardia.
What is the most likely diagnosis?
A. Pulmonary embolism
B. Sepsis
C. Fat embolism
D. Wound infection

The keywords to differentiate between PE and FE: CNS symptoms and a petechial
rash are found in FE.
Urology
Testicular torsion -> less than 12h with horizontal high riding
testis, and absent cremastric reflex. Severely tender testis.
Urgent surgical exploration.

Appendicular torsion -> typically less than 1 day with


vertical/longitudinal, blue dots sign, and upper pole
tenderness. Decreased vascularity on US.

Epididymitis -> typically more than 1 day, edematous, with


an increase in vascularity on US.

Incarcerated inguinal hernia -> unilateral swelling which


extends to the inguinal region, and the testis cannot be
palpated.

Hydrocele -> positive transillumination test and no pain.

Trauma
Intraperitonal injury -> surgical exploration
Extraperitoneal injury -> catheter repair then reassess 2
weeks later
Urethral injury -> suprapubic catheter, if not available
choose retrograde urethrogram to diagnose. Foley’s is C/I in
urethral injuries.

The most significant risk factor for malignancy in a patient


with BPH is age.

Undescended testis
Palpable under general anesthesia? Go for orchidopexy.
Still not palpable? Go for diagnostic laparoscopy.

Next step
Gross hematuria -> Urine cytology
Microscopic hematuria -> Repeat urinalysis

Elderly painless hematuria -> Cystoscopy


Elderly painful micturition -> Foley’s catheter
312- Case suspected to have renal stone, what’s the best diagnostic tool:
A. KUB X Ray
B. Ultrasound
C. CT CT without contrast is both initial and the best. Radiolucent stones
D. MRI do not show up on an x-ray so we can't rely on it.

313- 4. Pt with back pain and groins his pain is severe that he cannot stay still and keep
rolling, he has hematuria urine analysis has blood and epithelial cells " no other
investigations” , whats the diagnosis?
- uretre stone
- appendicitis
- pancreatitis
- cholecystitis

314- Patiet with uretric stone measure 4mm in US . Stable


Management
1- double J stent
2- conservative
3- ESWL Small stones pass spontaneously and need no
intervention.

315- A child has scrotal pain since 1 day, on exploration the cord was edematous and
inflamed with red right hemiscrotum, what is the diagnosis?
A. Testicular torsion
B. Inguinal hernia
C. Testicular appendages torsion
D. Epididymoorchitis
316- 14 years old came with sever sudden scrotal pain and on examination high riding
and sever tenderness on palpate.. dx?
A. Testicular torsion
B. Orchitis Another recall: What to do next? Urgent surgical exploration.
C. Hydrocele

317- 12 y boy presented with 2 days of tenderness in upper pole of right testis with
right scrotal inflammation (redness …),the testis is in longitudinal position (no high
riding testis) diagnosis?
A-Testicular torsion
B-Appendicular torsion
C-Cry..orchitis
D- Hydrocele

318- Child came with painfull red swollen hemiscrotum. On examination, mass was
palpated with -ve cough impulse. The mass was tender and extended to the inguinal
area. Left testes cannot be palpable. Which of the following is the most likely
diagnosis?
A- testicular torsion
B- Epidydomorchitis
C- Incarcerated inguinal hernia
D- Testicular appendicular torsion
319- Male with enlarged non painful scrotum. Tranllumination test was positive
hydrocele
Cyctocyle

320- Patient with pelvic fracture with extra peritoneal bladder injury, what is the
appropriate management
A. Suprapubic catheterization
B. Catheter Drainage for 2 weeks then reassess
C. Catheter drainage for 2 weeks then repair
321- Pelvic Fracture with injury to membranous part of urethra. Blood seen in external
meatus. What is the most appropriate action?
A. Cystoscopy
B. Folly’s Catheter
C. Suprapubic catheter Uretheral injury? Superpubic catheter. As for foley's it's C/I.
D. CT pelvis
322- post MVA with urethral injury , what next : (i’m sure no suprapupic tube in choices)
cystoscopy
folly catheter
retrograde urthrogram
fluoroscopic cystogram

323- 18 months child with left Undescended testes not palpable in the inguinal region, left
one is there what’s
the most appropriate to do?
A. Left orchidopexy
B. Diagnostic laparoscopy
C. Wait till 3 years
324- Young male baby came to well-baby clinic, upon examination his right testis was
palpable in the inguinal
canal and small in size and easily moved to scrotum, the left is normal, what is the cause?
A. Ectopic testis
B. Undescended testis Palpable in the inguinal canal but easily moved back to the
C. Testicular torsion scrotum.
D. Retractile testis

325- 62 years old male with Benign Prostatic Hyperplasia (BPH), His BMI: 41, what is the risk
of BPH in this case?
A. Age
B. Obesity
326- 67 years old male complaining of painless hematuria , whx is the diagnostic test?
A- Cystoscopy
B- CT abdomen
C- Iv pyelogram
D- Us
327- 82 year old male with very painful micturition what is the most approppriate
management
A- Abx for uti
B- Foley catheter
C- Cysoscopy and turp

328- 7year-old boy came with his parents complaining of asymptomatic hematuria.
Otherwise the patient is healthy. Urinalysis showed: RBSs, which of the following is the
most appropriate next step?

repeat urinalysis
Urine cytology
Renal Biopsy Visible (gross) hematuria.
Cystoscopy

329- 6 years old child and came becuae shie found accidently to have RBC in urine during
routine urine analysis Mother said she was having exercise yesterday She looks healthy
otherwise Best action
A-Ask for serum creatinine
B- Do Serum albumin
C-repeat urine analysis in few days
D-Send her for biopsy

Exercise induced microscopic hematuria.


Below are some repeated questions.
330- A 65-year-old heavy smoker is coming for a general examination.. What is the best
screening test for him?
A. Osteoporosis
B. Colon cancer Grade A (colon cancer) vs Grade B (AAA).
C. AAA

331- Patient with Cushing Syndrome with proven Right Adrenal Adenoma on MRI for
resection, what is the
peri operative management?
A. Hydrocortisone
B. Fludrocortisone
332- Which of the following is true in necrotizing pancreatitis
A) increased lipolysis
B) Hypoglycemia Always go for CT scan to diagnose.
C) Decreased gluconeogenesis
333- Which of the following indicates compensated shock?
A. Anuria
B. Confusion
C. Hypotension
D. pale peripheries
334- Trauma patient, currently well. lucid interval reported by paramedics, now he is
deteriorating. Most likely
diagnosis?
A. Epidural hematoma
B. Subdural hematoma
C. Base of skull fracture
335- Old patient with altered lvl of consiousness fell from the stairs, was brought to ER,
done CT for him showing epidural hematoma. Asking about most likely affected artery:
A. Pontine
B. Basilar
C. Anterior cerebral artery
D. Middle meningeal artery

336- Patient had surgery after 1 day he had diminished breath sounds in the left lower lobe
how to manage?
A- Incentive spirometry
B-Chest tube Case of atelectasis.
C-Needle decompression
337- Elderly female that has back, and she is stooping and bending her back while she
walks to relieve the
pain. What is the diagnosis?
A. Lumbar spine stenosis
Physiotherapy next, spinal leminectomy best.
B. Degenerative Lumbar spine

338- A 56-year-old woman was exposed to chemical burns to her


trunk. on arrival to the Emergency Department, she was
conscious and alert. Examination showed 5 cm x 11 cm wound at her back with burned
clothes and powder of chemicals on her clothes and skin
Which of the following is most appropriate next step after removing her clothes?
A. Antibiotic.
B. Powder sweeping
C. Surgical debridment.
D. Water irrigation for 30 minutes.
339- Burn patient, resuscitation done, which of the following reflect a good
resuscitation has been achieved?
A- normalization of heart rate
B- normalization of blood pressure
C- Urine out of 0.6ml/kg/h
D- Central venous pressure 12

340- Pt 28 with bleeding during defecation for two month


In protoscop shows multiple polys , ur diagnosis ?
A. Diverticulosis
B. Familia polyp
C. Crohns

341- Child presents with hip and groin pain. An US was done to reveal hip joint effusion.
He is afebrile. What
is the most likely diagnosis?
A. Toxic synovitis
B. Benign acute myositis
C. Osteomyelitis
D. Septic arthritis

In the full recall the patient didn't have high WBC.


342- old patient came with subdural hematoma with signs of lateralization imaging
revealed 13 mm shifting. his GCS 7/15 then was intubated and resuscitated what to do
next
A- iv mannitol
B- elevate head of bed
C- hyperventilate
D- urgent craniotomy

343- Pediatric patient had a fall from 1 story high and direct trauma to the head, presents
with hemotympanium No loss of consciousness, no vomiting , neuro exam Normal Ear :
Ruptured tympanic membrane with intact external auditory canal Most likely bone
fracture:
A- Mastoid
B- Maxillary
C- Basal skull
344- Pt with face laceration ,repair was done and they use lidocaine Most common
complications of lidocaine?
A-Nystagmus
B-Ventricular tachycardia
C-Drowsiness
345- Patient had liver laceration and the patient is hemodynamically unstable. What to
do?
A. Right hepatectomy
B. Perihepatic packing
C. Right hepatic artery ligation
D. Ligation of the involved vessel
346- 12- year-old received a nonspecific blunt trauma on his abdomen and later presented
with generalized
abdominal pain. Imaging of the spleen showed a 7 mm hematoma and 4 cm tear (grade 3).
Your
management is?
A. Splenectomy
B. Spleen preserving surgery
C. Conservative
Only go with splenectomy if the patient is unstable.

347- Patient underwent left lower parathyroidectomy for primary hyperparathyroidism


(adenoma). He
presented 4 months later with depressed mood and fatigability. Both parathyroid hormone
and calcium
were high, what is the most common cause?
A. Parathyroid hyperplasia
B. Missed adenoma
C. New adenoma
D. Parathyroid cancer

Less than 6 months -> missed adenoma.


More than 6 months -> parathyroid hyperplasia.
348- 40Y/O female with 12-year history of varicosities, complaining of LL edema and
heaviness when standing. Distal pulses are intact, and there is bilateral varicose in the great
saphenous vein territory. What is your management?
A Sclerotherapy
B Endovenous laser
C Thermal ablation

349- Patient fell on outstretched hand, pain at anatomical sniff box, which bone fracture is
it?
A. Colles
B. Scaphoid
350- The World Health Organization (WHO) published the WHO Surgical Safety Checklist in
2008 in order to
increase the safety of patients undergoing surgery. The Checklist consists of three phases
of surgical
procedure:
A. Before admission, Before skin incision, at discharge from hospital.
B. Before admission, before induction of anesthesia, at discharge from hospital.
C. Before induction of anesthesia, Before skin incision, Before patient leaves operating room.
D. Before admission, Before induction of anesthesia, Before patient leaves operating room.

You might also like